Vous êtes sur la page 1sur 104

Polycopié

COURS D'ALGEBRE 4:
Formes bilinéaires et réduction
des formes quadratiques

BOUDELIOU AMMAR
ET
MATMAT CHAHRAZADE

Département de Mathématiques
Faculté de sciences exactes
Université Frères Mentouri,
Constantine 1

©- 2017 A. Boudeliou, C. Matmat


République Algérienne Démocratique et Populaire
Ministère de L’Enseignement Supérieur et de la Recherche Scientifique

Université Des Frères Mentouri, Constantine1


Faculté Des Sciences Exactes
Département De Mathématiques

Polycopié

COURS D'ALGEBRE 4:
"Formes bilinéaires et réduction des
formes quadratiques "

Par:
BOUDELIOU AMMAR
et
MATMAT CHAHRAZADE

©- 2017 A. Boudeliou, C. Matmat


Table des matières

I FORMES LINÉAIRES ET DUALITÉ . . . . . . . . . . . . . . . . . 5

1.1 Formes linéaires, espace dual . . . . . . . . . . . . . . . . . . . . . . 5

1.2 Hyperplans . . . . . . . . . . . . . . . . . . . . . . . . . . . . . . . . 6

1.3 Base duale . . . . . . . . . . . . . . . . . . . . . . . . . . . . . . . . 8

1.4 Bidual d’un espace vectoriel . . . . . . . . . . . . . . . . . . . . . . 12

II FORMES BILINÉAIRES SUR UN ESPACE VECTORIEL DE DI-

MENSION FINIE . . . . . . . . . . . . . . . . . . . . . . . . . . . . . 18

2.1 Dé…nitions . . . . . . . . . . . . . . . . . . . . . . . . . . . . . . . . 18

2.2 Matrice associée à une forme bilinéaire . . . . . . . . . . . . . . . . . 21

2.3 Changement de base . . . . . . . . . . . . . . . . . . . . . . . . . . . 24

2.4 Noyau et rang d’une forme bilinéaire . . . . . . . . . . . . . . . . . . 26

2.5 L’équivalence entre formes bilinéaires . . . . . . . . . . . . . . . . . 27

2.6 Orthogonalité . . . . . . . . . . . . . . . . . . . . . . . . . . . . . . 28

III ESPACES EUCLIDIENS . . . . . . . . . . . . . . . . . . . . . . . . . 36

3.1 Produit scalaire . . . . . . . . . . . . . . . . . . . . . . . . . . . . . 36

3.2 Espace euclidien . . . . . . . . . . . . . . . . . . . . . . . . . . . . . 36

3.3 Orthogonalité . . . . . . . . . . . . . . . . . . . . . . . . . . . . . . 42

3.4 Matrices orthogonales . . . . . . . . . . . . . . . . . . . . . . . . . . 46

1
3.5 Diagonalisation des matrices symétriques réelles . . . . . . . . . . . 48

3.5.1 Adjoint d’un endomorphisme dans un espace euclidien . . . . 49

3.5.2 Endomorphisme auto-adjoint . . . . . . . . . . . . . . . . . . 52

3.5.3 Diagonalisation des matrices symétriques . . . . . . . . . . . 53

IV FORMES QUADRATIQUES . . . . . . . . . . . . . . . . . . . . . . 59

4.1 Formes quadratiques . . . . . . . . . . . . . . . . . . . . . . . . . . . 59

4.2 Matrice d’une forme quadratique . . . . . . . . . . . . . . . . . . . . 64

4.3 Réduction des formes quadratiques . . . . . . . . . . . . . . . . . . . 66

4.3.1 Réduction dans le cas général . . . . . . . . . . . . . . . . . . 66

4.3.2 Réduction en carrés de Gauss . . . . . . . . . . . . . . . . . . 70

4.3.3 La réduction dans une base de vecteurs propres . . . . . . . . 76

4.3.4 Formes quadratiques équivalentes . . . . . . . . . . . . . . . 80

V FORMES HERMITIENNES . . . . . . . . . . . . . . . . . . . . . . . 83

5.1 Rappels sur les nombres complexes . . . . . . . . . . . . . . . . . . . 83

5.2 Formes hermitiennes . . . . . . . . . . . . . . . . . . . . . . . . . . . 84

2
Préface
L’ouvrage que nous présentons ici peut servir comme support d’algèbre bilinéaire

destiné aux étudiants de la license LMD, en particulier pour les étudiants de 2ème

année (Maths ) et il peut aussi servir aux étudiants des sciences et technologie. Depuis

quelques années, l’algèbre linéaire et bilinéaire est devenue une partie essentielle du

bagage mathématique nécessaire aux ingénieurs, physiciens et autres scienti…ques.

Ce besoin re‡ète l’importance et les applications étendues du sujet. Ce polycopié est

destiné à être utilisé comme manuel pour un cours d’algèbre bilinéaire ou comme

supplément à d’autres ouvrages. Il vise à présenter des notions de base de l’algèbre

bilinéaire qui sera utile à tous les lecteurs quelle que soit leur spécialisation. Il est

inclus plus de matière que l’on en peut insérer dans la plupart des cours d’algèbre

bilinéaire. Ceci a été fait dans le but de rendre l’ouvrage plus souple, de fournir un

livre de référence utile et stimuler l’intérêt porté à cette matière.

Chaque chapitre comprend des énoncés clairs de dé…nitions de principes et de

théorèmes avec démonstrations, des éléments d’illustration et de description et des

exemples qui servent à illustrer et à ampli…er la théorie, à mettre au point de façon

précise les passages délicats, sans lesquels l’étudiant se sent constamment sur un

terrain incertain, et à permettre la répétition des principes fondamentaux, si essentiels

à une étude e¢ cace. Cet ouvrage est enrichi par une biographie de quelques savants

cités dans le cours.

Le premier chapitre est consacré à la présentation de di¤érentes notions sur les

formes linéaires et dualité. le deuxième chapitre et consacré à l’étude des formes bili-

néaires dé…nies sur un espace vectoriel de dimension …nie où on donne des dé…nitions,

la matrices d’une forme bilinéaire, changement de base, noyau et rang d’une forme bi-

linéaire, équivalence entre formes bilinéaires et orthogonalité par rapport à une forme

bilinéaire. Dans le troisième chapitre on présente des notions essentielles concernant :

le produit scalaire, les espaces euclidiens, l’orthogonalité, les matrices orthogonales

3
et la diagonalisation des matrices symétriques réelles. En quatrième chapitre nous

nous intéressons à l’étude des formes quadratiques et ses réduction qui est notre but

principal de ce polycopié. On termine notre polycopié par la donnée des notions de

bases sur les formes hermitiennes.

Le contenu de cet ouvrage est le fruit des Cours d’algèbre 4 que nous enseignons

en deuxième année au département de mathématiques.

Dr. A. BOUDELIOU

4
Chapitre I

FORMES LINÉAIRES ET DUALITÉ

1.1 Formes linéaires, espace dual


Dans tout ce chapitre K désignera un corps commutatif et E un K-espace vectoriel

(de dimension …nie ou non).

Dé…nition 1.1 Soit E un K espace vectoriel. On appelle forme linéaire sur E

toute application linéaire de E dans K.

On appelle espace dual de E, noté E , l’espace vectoriel des formes linéaires

sur E. Autrement dit, E = L (E; K) et ' 2 E signi…e que ' : E ! K est une

application linéaire telle que : 8 (x; y) 2 E 2 et 8 ( ; ) 2 K 2 ; ' ( x + y) = ' (x) +

' (y) :

Exemple 1.2 (a) L’application R2 ! R; (x; y) 7! 2x + y est une forme linéaire sur

R2 :

(b) L’application E ! K; x 7! 0 est une forme linéaire, appelée forme nulle sur

E:

(c) Si E = K [X] l’espace des polynômes à coe¢ cients dans K; alors pour tout

a 2 K; l’application P 7! P (a) est une forme linéaire sur E:

(d) Si E = C ([a; b] ; R) l’espace des fonctions réelles continues sur [a; b], alors
Zb
l’application f 7! f (t)dt est une forme linéaire sur E:
a Pn
(e) Si E = Mn (K) ; alors l’application trace : A = (aij ) 7! tr (A) = 1 aii est

une forme linéaire sur E:

(f ) Soit E un K espace vectoriel de dimension …nie n et B = (e1 ; :::; en ) une base


P
de E: Tout élément x 2 E s’écit d’une manière unique sous la forme x = n1 xi ei .

5
Pour chaque j 2 [1; n] ; l’application ej : E ! K; x 7! ej (x) = xj est une forme

linéaire sur E; appelée j eme forme coordonnée relative à la base B.

D’une façon générale on a :

Proposition 1.3 Soit n 2 N :

(a) Soit ( 1 ; :::; 2 K n : L’application de K n dans K qui à tout x = (x1 ; :::; xn ) 2


n)
P
K n associe le scalaire '(x) = n1 j xj ; est une forme linéaire sur K n :

(b) Réciproquement, pour toute forme linéaire ' sur K n ; il existe un n uplet
P
( 1 ; :::; n ) 2 K n tel que pour tout x = (x1 ; :::; xn ) 2 K n ; on ait '(x) = n1 j xj :

Preuve. Le point (a) résulte d’une véri…cation directe.

(b) Soit (e1 ; :::; en ) la base canonique de K n et soit ' : K n ! K une forme linéaire

sur K n :
Pn
Tout élément x 2 K n s’écrit d’une façon unique sous la forme x = 1 xj ej et
P
donc '(x) = n1 xj ' (ej ) : D’où l’existence et l’unicité des j = ' (ej ) ; 1 j n:

Proposition 1.4 Soit E un K espace vectoriel de dimension …nie. Alors son dual

E est de dimension …nie et dim E = dim E .

Preuve. En e¤et, dim E = dim L (E; K) = dim E dim K = dim E:

1.2 Hyperplans
Dé…nition 1.5 Soit E un K espace vectoriel. On appel hyperplan de E, le noyau

de toute forme linéaire sur E autre que la forme nulle.

Autrement dit, une partie H de E est un hyperplan de E s’il existe ' 2 E n f0g

telle que H = ker (') : On dit alors que la relation '(x) = 0 est une équation de

l’hyperplan H.

Exemple 1.6 (a) H = fA 2 Mn (K) ; T r (A) = 0g est un hyperplan de Mn (K) :

(b) H = f(x; y; z) 2 R3 ; 2x 3y + z = 0g est un hyperplan de R3 :

(c) H = fP 2 K [X] ; P (0) = 0g est un hyperplan de K [X] :

6
Proposition 1.7 Soit H un sous-espace vectoriel de E. Les propriétés suivantes sont

équivalentes :

(a) H est un hyperplan de E.

(b) Il existe dans E une droite vectorielle D supplémentaire de H telle que

E=H D:

Si E est de dimension …nie, les conditions précédentes sont équivalentes à

(c) dim(H) = dim(E) 1 (autrement dit, H est de codimension 1).

(d) Toute droite vectorielle de E engendrée par un vecteur n’appartenant pas à H

est un supplémentaire de H.

Preuve. (a) ) (b) : Si H est un hyperplan de E, il existe ' 2 E n f0g telle que

H = ker (') : Puisque ' n’est pas nulle, il existe v 2 E tel que '(v) 6= 0: Considérons

la droite vectorielle D = Kv et montrons que E = H D: Soit x 2 H \ D: Il existe

2 K tel que x = v et '(x) = 0, donc '(v) = '( v) = 0: Comme '(v) est non

nul, on déduit que = 0 et x = 0. Ainsi H \ D = f0g. Soit x 2 E et montrons que


'(x) '(x)
x 2 H + D. Soit = '(v)
et posons y = x '(v)
v qui est un élément de H puisque
'(x) '(x)
' (y) = ' x '(v)
v = '(x) '(v)
' (v) = 0: D’où x = y + v 2 H + D.

(c) Si E est de dimension …nie, on a dim ker (') + dim Im (') = dim E. D’où

dim H + dim K = dim E; alors dim(H) = dim(E) 1 (puisque dim K = 1).

Remarque 1.8 Si E est de dimension …nie n et B = (ej )nj=1 une base de E. Rela-

tivement à la base B un hyperplan H de E admet une équation unique, de la forme

a1 x1 + a2 x2 + ::: + an xn = 0 où on a noté x1 ; :::; xn les coordonnées des vecteurs x 2 E

par rapport à B.

Corollaire 1.9 Deux formes linéaires non nulles sur un espace vectoriel E sont pro-

portionnelles si et seulement si elles ont le même noyau.

Preuve. Soit '; 2 E n f0g : Supposons que ker (') = ker ( ) = H: Soit v 2
=H
'(v)
(donc '(v) et (v) ne sont pas nuls). Posons = (v)
et montrons que ' = :

7
Soit x 2 E, d’après la proposition précédente E = H Kv; donc x s’écrit x =

y + v avec y 2 H et 2 K: D’où '(x) = ' (y) + '(v) = '(v) = ( (v)) =

( (y) + (v)) = (x) : Donc ' et sont proportionnelles.

La réciproque est immédiate car

ker(') = fx 2 E : '(x) = 0g

= fx 2 E : (x) = 0; 6= 0g

= fx 2 E : (x) = 0g

= ker ( ) :

1.3 Base duale


Soit E un espace vectoriel de dimension …nie, n 1. Nous nous renvoyons à

l’exemple 1.2 pour la dé…nition des formes linéaires ei :

Proposition 1.10 Soit B = (ej )nj=1 une base de E. La famille des formes coordon-

nées B = (ei )ni=1 est une base de l’espace dual E , appelée base duale de B.

De plus, pour tout i; j 2 [1; n], on a les relations (d’orthogonalité) de Kronecker :


8
>
< 1 si i = j
ei (ej ) = ij =
>
: 0 si i 6= j

Preuve. Par dé…nition

X
n
ei : E ! K; x = xj ej 7! ei (x) = xi :
1

Donc ei (ej ) = ij pour tout i; j 2 [1; n] :


P
Soit ' 2 E et considérons la forme linéaire = ni=1 ' (ei ) ei : Pour tout j 2
P
[1; n], on a (ej ) = ni=1 ' (ei ) ei (ej ) = ' (ej ) : Les formes linéaires ' et coïncident

sur une base de E sont donc égales.

8
Par conséquent,
X
n
'= ' (ei ) ei ;
i=1

et la famille B est une famille génératrice de E . Comme dim E = dim E = n; la

famille B est une base de E .

Corollaire 1.11 Soient B = (ej )nj=1 une base de E et B = (ei )ni=1 sa base duale,

alors on a les relations suivantes :


P
I 8x 2 E; x = ni=1 ei (x) ei :
P
I 8' 2 E ; ' = ni=1 ' (ei ) ei :

I 8f 2 L (E) ; aij = ei (f (ej )) ; où (aij )1 i;j n = M atB (f ) :

Proposition 1.12 (a) Si ' est une forme linéaire non nulle sur E, il existe un

vecteur x 2 E (non nul) tel que '(x) = 1.

(b) Si x est un vecteur de E non nul, il existe une forme linéaire ' 2 E telle

que '(x) = 1.

Preuve. (a) Si ' 2 E n f0g, alors il existe v 2 E tel que '(v) 6= 0. Le vecteur
v
x= '(v)
convient.
P
(b) Si x = ni=1 xi ei 2 E est non nul, alors il existe i0 2 [1; n] tel que ei0 (x) =
1
xi0 6= 0: La forme linéaire ' = e
ei (x) i0
convient.
0

Proposition 1.13 Toute base de E est la base duale d’une unique base de E, appelée

base préduale.

Preuve. Soit F = ('1 ; :::; 'n ) une base de E . L’application : x 7! ('1 (x); :::; 'n (x))

de E dans K n est linéaire. Soit x 2 ker( ), donc '1 (x) = ::: = 'n (x) = 0: Si x n’est

pas nul, alors d’après la proposition précédente, il existe ' 2 E telle que '(x) = 1.
P
Cette forme linéaire s’écrit dans la base F sous la forme ' = ni=1 i 'i . Par consé-
P
quent 1 = '(x) = ni=1 i 'i (x) = 0 ce qui est absurde. On en déduit que x = 0, que

9
le noyau de est réduit à f0g et que est injective. Comme dim (E) = n = dim(K n ),

l’application est un isomorphisme.

Notons (e1 ; :::; en ) la base canonique de K n . Pour tout j, un vecteur ej véri…e

'i (ej ) = ij pour tout i si, et seulement si (ej ) = ej . Puisque est un isomorphisme,
1 1
la famille B = ( (e1 ) ; :::; (en )) est une base de E et c’est la seule famille de E

satisfaisant aux conditions de Kronecker. Par conséquent, B est l’unique base de E

dont F est la base duale.

Proposition 1.14 (Changement de base duale)

Soient B1 et B2 deux bases de E, et soit P la matrice de passage de B1 à B2 .

Alors la matrice de passage de B1 à B2 est t P 1


.

Preuve. Posons B1 = (e1 ; :::; en ) ; B2 = (f1 ; :::; fn ) et P = (aij )1 i;j n ; puis notons

Q = (bij )1 la matrice de passage de B1 à B2 : Par dé…nition de la matrice de


i;j n
Pn
passage on a pour tout k 2 [1; n] ; fk = l=1 alk el et pour tout j 2 [1; n] ; fj =
Pn
l=1 bij ei : Donc pour tout j; k 2 [1; n] ;
! n !
X
n X
jk = fj (fk ) = bij ei alk el
l=1 l=1
n X
X n
= bij alk il
l=1 l=1
X
n
= bij aik
i=1
t
= QP jk
:

Donc t QP = In et Q =t P 1
:

Corollaire 1.15 (Calcul pratique de la base duale)

Soient B0 = (ei )ni=1 la base canonique de E et B 0 =(ei )ni=1 sa base duale. Soit

B = (vi )ni=1 une autre base de E et B = (vi )ni=1 sa base duale. Les vecteurs vi

(respectivement vi ) étant exprimés dans la base B0 (respectivement B 0 ). Alors

1
M atB0 (B ) =t (M atB0 (B)) :

10
Exemple 1.16 (a) Soient les vecteurs v1 = ( 3; 1; 1); v2 = (5; 2; 1); v3 = (6; 2; 1)

de R3 exprimés dans la base canonique


0 (e1 ; e2 ; e3 ). La1famille B = (v1 ; v2 ; v3 ) est une
B 3 5 6 C
B C
base de R3 , puisque la matrice P = B
B 1 2 2 C C est inversible (de déterminant
@ A
1 1 1
1). Déterminons sa base duale.

Soit B = (v1 ; v2 ; v3 ) la base duale de B. Alors la matrice de passage de B 0 =(e1 ; e2 ; e3 )(base

duale de la base canonique) à B est


0 1
B 0 1 1 C
B C
t
P 1
=B
B 1 3 2 C
C:
@ A
2 0 1

On conclue donc que 8


>
> v1 (x; y; z) = y + 2z
>
>
<
> v2 (x; y; z) = x + 3y ;
>
>
>
: v (x; y; z) = x 2y + z
3

ou encore 8
>
> v1 = e2 + 2e3
>
>
<
> v2 = e1 + 3e2 :
>
>
>
: v =e
3 1 2e2 + e3
(b) Soient 8
>
> f1 (x; y; z) = x + 2y + 3z
>
>
<
> f2 (x; y; z) = 2x + 3y + 4z
>
>
>
: f3 (x; y; z) = 3x + 4y + 6z

trois formes linéaires sur R3 . Dans la base B 0 =(e1 ; e2 ; e3 ) elles s’écrivent


8
>
> f1 = e1 + 2e2 + 3e3
>
>
<
> f2 = 2e1 + 3e2 + 4e3 :
>
>
>
: f3 = 3e + 4e + 6e
1 2 3

11
La famille F = (f1 ; f2 ; f3 ) est bien une base de (R3 ) puisque la matrice
0 1
B 1 2 3 C
B C
Q=B B 2 3 4 C
C
@ A
3 4 6

est inversible. Soit B = (v1 ; v2 ; v3 ) la base de R3 dont F est la base duale. La matrice

de passage de de B0 à B est donc


0 1
B 2 0 1 C
B C
t
Q 1
=B
B 0 3 2 C
C:
@ A
1 2 1

On conclue donc que v1 = ( 2; 0; 1); v2 = (0; 3; 2) et v3 = (1; 2; 1).

1.4 Bidual d’un espace vectoriel


Dé…nition 1.17 Soit E un espace vectoriel. Le dual de E , noté E est appelé bi-

dual de E.

Proposition 1.18 Soit E un K-espace vectoriel de dimension …nie et considérons

l’application
:E!E

x 7! x : E ! K

' 7! '(x)
est un isomorphisme d’espaces vectoriels.

Preuve. En e¤et, la linéarité est facile à démontrer car : 8x; y 2 E; 8 ; 2

K; ( x + y) = ( ^
x + y) : E ! K. D’autre part, pour tout ' 2 E on a

( ^
x + y) (') = ' ( x + y) = ' (x) + ' (y) = x (') + y (') = x + y (') :

D’où

( x + y) = ( ^
x + y) = x + y = (x) + (y) :

12
La bijection : Soit x 2 Ker( ), alors '(x) = 0 pour tout ' 2 E . On en déduit

d’après la proposition 1.12 que x = 0. Donc est injectif et comme E, E et E ont

la même dimension, est un isomorphisme de E sur E .

Cet isomorphisme permet d’identi…er le bidual E à E.

Exercise 1.19 Soit E = R3 [X] l’espace des polynômes à coe¢ cients réels et de degré

inférieur ou égale à 3: Pour P 2 R3 [X] on pose

f1 (P ) = P (0); f2 (P ) = P (1); f3 (P ) = P 0 (0); f4 (P ) = P 0 (1)

1. Montrer que fi est une forme linéaire sur E pour i = 1; 2; 3; 4 et que ff1 ; f2 ; f3 ; f4 g

est une base du dual E de E:

2. Déterminer une base fP1 ; P2 ; P3 ; P4 g de E dont ff1 ; f2 ; f3 ; f4 g est la base duale.

Solution :

1. Pour P; Q 2 E; 2 R et x 2 R, on a

(P + Q) (x) = P (x) + Q(x);

alors f1 et f2 sont des formes linéaies sur E, et

(P + Q)0 (x) = P 0 (x) + Q0 (x);

donc f3 et f4 sont des formes linéaies sur E.

Soit i pour i = 1; 2; 3; 4 des réels tels que

1 f1 + 2 f2 + 3 f3 + 4 f4 =0

c’est-à-dire pour tout P 2 E

0 0
1 P (0) + 2 P (1) + 3P (0) + 4P (1) = 0:

13
Prenant successivement P (x) = 1; x; x2 et x3 on obtient les relations :

1 + 2 = 0

2 + 3 + 4 = 0

2 +2 4 = 0

2 +3 4 = 0:

Les deux dernières équations assurent que 4 = 0 puis 2 = 0: La deuxième assure

que 3 = 0; puis la première relation assure que 1 = 0:

Ainsi les quatre formes linéaires f1 ; f2 ; f3 ; f4 sont linéairement indépendantes dans

E ; et comme dim E = dim E = 4; elles forment une base de E :

2. On cherche une base fP1 ; P2 ; P3 ; P4 g de E telle que pour i; j = 1; 2; 3; 4


8
>
< 1; i = j
fi (Pj ) = ij =
>
: 0; i 6= j

On doit avoir

f1 (P4 ) = f2 (P4 ) = f3 (P4 ) = 0 et f4 (P4 ) = 1

c’est-à-dire

P4 (0) = P4 (1) = P40 (0) = 0 et P40 (1) = 1:

Ainsi P4 s’annule en 0 et 1 et P40 s’annule en 0, donc le polynôme P4 de degré 3

s’annule à l’ordre 2 en 0 et l’ordre 1 en 1; il est donc de la forme

P4 (x) = a4 x2 (x 1) :

En particulier

P40 (x) = a4 2x (x 1) + a4 x2

= a4 3x2 2x ;

et comme P40 (1) = 1, alors a4 = 1:

14
Ainsi

P4 (x) = x2 (x 1) :

Ensuite on doit avoir

f1 (P3 ) = f2 (P3 ) = f4 (P3 ) = 0 et f3 (P3 ) = 1;

c’est-à-dire

P3 (0) = P3 (1) = P30 (1) = 0 et P30 (0) = 1:

Ainsi P3 s’annule en 0 et 1 et P30 s’annule en 1, donc le polynôme P3 de degré 3

s’annule à l’ordre 2 en 1 et l’ordre 1 en 0; il est donc de la forme

P3 (x) = a3 x (x 1)2 :

En particulier

P30 (x) = a3 (x 1)2 + 2a3 x (x 1)

= a3 (x 1) (3x 1) ;

et comme P30 (0) = 1, alors a3 = 1:

Ainsi

P3 (x) = x (x 1)2 :

De même on calcule P1 et P2 on trouve

P1 (x) = (2x + 1) (x 1)2

P2 (x) = x2 ( 2x + 3) :

Exercise 1.20 Soit E = R4 ; et

F = f(x; y; z; t) 2 E j x + y z + t = 0g

et

D = vect (v) ; v = (1; 1; 1; 1):

15
a) Montrer que F est un hyperplan de E:

b) Montrer que F et D sont deux sous-espaces supplémentaires de E:

c) Soit un réel m 2 R et u = (m; m + 1; 2m; m 2) 2 E: Pour quelles valeurs de

m les sous-espaces F et = vect (u) sont-ils supplémentaires dans E ?

Solution :

a) Solution. Soit ' : R4 ! R dé…nie par '(x; y; z; t) = x + y z + t, alors ' est

linéaire (elle s’écrit :


0 1 0 1 0 1
x x x
B C B C B C
B C B C B C
B y C B y C B y C
B C B C B C
'B C = 1 1 1 1 B C = AX où A = 1 1 1 1 et X = B C ).
B C B C B C
B z C B z C B z C
@ A @ A @ A
t t t

Puisque ' 6= 0 (car '(1; 0; 0; 0) = 1 6= 0) et H = ker ('), l’ensemble H est un

hyperplan de E.

b) Puisque '(v) = 2 6= 0, alors v 2


= H d’où F D = E.

c) Si u 2
= F; alors F = E. Or

u2F ,m+m+1 2m + m 2=m 1 = 0 , m = 1:

Pour m 6= 1, on a F = E. Si m = 1, le générateur de est dans F donc F et

ainsi F \ = 6= f0g ; la somme n’est pas directe, ils ne sont pas supplémentaires.

Exercise 1.21 Soit E = Rn [X] muni de la base B = f1; X; : : : ; X n g. Pour tout i de

f0; : : : ; ng, on dé…nit une forme linéaire fi sur E par


8
>
< 1 si i = j
j
8j 2 f0; : : : ; ng; fi (X ) =
>
: 0 si i 6= j

1. Démontrer que (f0 ; : : : ; fn ) est une base de E .

16
2. On considère les deux éléments et de E dé…nis par : 8P 2 E; (P ) = P (1)

et (P ) = P 0 (0). Déterminer les coordonnées de chacune des formes et dans la

base (f0 ; : : : ; fn ).

3. Rn 1 [X] est-il un hyperplan de Rn [X] ? Si oui, en donner tous ses supplémen-

taires.

4. Rn 1
-est-il un hyperplan de Rn ? En déduire une caractérisation de ses hyper-

plans.

17
Chapitre II

FORMES BILINÉAIRES SUR UN ESPACE

VECTORIEL DE DIMENSION FINIE

Dans tout ce chapitre K désignera un corps commutatif et E un K-espace vectoriel

de dimension …nie.

2.1 Dé…nitions
Dé…nition 2.1 Une application

b:E E!K

(x; y) 7! b(x; y)

est dite forme bilinéaire lorsqu’elle est linéaire par rapport à chacune de ses variables,

c’est à dire

– Pour y 2 E …xé, l’application x 7! b(x; y) est linéaire ;

– Pour x 2 E …xé, l’application y 7! b(x; y) est linéaire.

Autrement dit, pour tout x; y; z 2 E; 1; 2; 1; 2 2K:

b( 1x + 1 y; z) = 1 b(x; z) + 1 b(y; z):

et

b(x; 2y + 2 z) = 2 b(x; y) + 2 b(x; z):

Dé…nition 2.2 Soit b : E E ! K une forme bilinéaire sur E. On dit que b est :

– Symétrique si pour tout x; y 2 E

b(x; y) = b(y; x):

18
– Antisymétrique si pour tout x; y 2 E

b(x; y) = b(y; x):

– Alternée si pour tout x 2 E

b(x; x) = 0:

Proposition 2.3 Toute forme bilinéaire alternée sur E est antisymétrique.

La réciproque est vraie si car(K) 6= 2. (car(K) =Caractéristique de K)

Preuve. Supposons que b est alternée, d’où on a

b(x + y; x + y) = b(x; y) + b(y; x) + b(x; x) + b(y; y);

d’où

b(x; y) + b(y; x) = b(x + y; x + y) b(x; x) b(y; y)

= 0 (car b est alternee):

Donc

b(x; y) = b(y; x):

Réciproquement, supposons que b est antisymétrique et car(K) 6= 2 d’où on a

2b(x; x) = b(x; x) + b(x; x)

= b(x; x) + b(x; x) (car b est antisymetrique)

= 0

Ceci implique que

b(x; x) = 0:

Notation 2.4 On note par L2 (E) l’ensemble des formes bilinéaires sur E et par

S2 (E) l’ensemble des formes bilinéaires symétriques sur E et par A2 (E) l’ensemble

des formes bilinéaires antisymétriques sur E.

19
Exemple 2.5 1. Soient E = R2 ; x = (x1 ; x2 ); y = (y1 ; y2 ) 2 E et soit l’application

b:E E ! R telle que

b(x; y) = x1 y1 + 5x2 y2 3x1 y2 3x2 y1 ;

b est une forme bilinéaire symétrique.

2. E = Rn , le produit scalaire canonique

X
n
hx; yi = xi yi ;
i=1

est une forme bilinéaire symétrique.

3. Soit E = C ([0; 1]; R) l’espace des applications continues de [0; 1] dans R,

b:E E!R
R1
(f; g) 7! b(f; g) = 0
f (t)g(t)dt

est une forme bilinéaire symetrique.

Proposition 2.6 L’ensemble L2 (E) muni de la lois intèrne (+) telle que :

(b1 + b2 )(x; y) = b1 (x; y) + b2 (x; y);

et de la multiplication par un scalaire (:) (lois extèrne) telle que

( :b)(x; y) = :b(x; y); 2K

est un K-espace vectoriel.

L’ensemble S2 (E) est un sous espace vectoriel de L2 (E).

Proposition 2.7 S2 (E) et A2 (E) sont deux supplémentaires de L2 (E): c-à-d. L2 (E) =

S2 (E) A2 (E):

Preuve. Pour tout b 2 S2 (E) \ A2 (E), on a pour tout x; y 2 E

b 2 S2 (E) et b 2 A2 (E) ) b(x; y) = b(y; x) et b(y; x) = b(x; y);

20
d’où

b(x; y) = 0;

donc b = 0, alors S2 (E) \ A2 (E) = f0g :

Pour tout b 2 L2 (E) et pout tout x; y 2 E on a


b(x; y) + b(y; x) b(x; y) b(y; x)
b(x; y) = +
2 2
= b1 (x; y) + b2 (x; y) :

On peut facilement véri…er que b1 2 S2 (E) et b2 2 A2 (E):

Finalement on déduit que L2 (E) = S2 (E) A2 (E):

2.2 Matrice associée à une forme bilinéaire


Soient E un K-espace vectoriel de dimension …nie n muni d’une base B = (e1 ; :::; en )

et b une forme bilinéaire sur E . Pour tout x; y 2 E on a


X
n X
n
x= xi ei ; y = yj ej :
i=1 j=1

Posons X; Y les matrices colonnes des coordonnées de x et y

2 3 2 3
6 x1 7 6 y1 7
6 .. 7 6 .. 7
X=6
6 . 7;Y = 6
7 6 . 7;
7
4 5 4 5
xn yn
d’où on a

Xn X
n
b(x; y) = b( xi ei ; yj ej )
i=1 j=1
X
n Xn
= xi b(ei ; yj ej ) (linearite par rapport a x)
i=1 j=1
Xn X
n
= xi yj b(ei ; ej ) (linearite par rapport a y)
i=1 j=1
XX
n n
= xi yj b(ei ; ej ):
i=1 j=1

21
Si on suppose b(ei ; ej ) = aij on obtient

X
n
b(x; y) = aij xi yj : (2.1)
i;j=1

La matrice M = [aij ]1 i;j n est appelée la matrice associée à la forme bilinéaire b:

L’expression (2.1) s’écrit sous la forme matricielle suivante


0 12 3
B a11 a1n C 6 y1 7
B . .. C 6 7
b(x; y) = x1 xn B .. ..
. . C 6 ... 7 ;
B C6 7
@ A4 5
an1 ann yn

d’où

b(x; y) =t XM Y:

Si b est symétrique alors

b(ei ; ej ) = b(ej ; ei ):

C.à.d.

aij = aji :

Donc M est une matrice symétrique.

Dans l’autre sens si M est une matrice symétrique dans Mn (K) alors l’application

b dé…nie par

b(x; y) =t XM Y;

telles que X; Y sont des matrices colonnes des coordonnées de x et y respectivement

dans la base B est une forme bilinéaire symétrique.


0 1
B 3 1 C
Exemple 2.8 1. La matrice M = @ A dé…nie sur R2 la forme bilinéaire
1 2
symétrique suivante

b(x; y) =t X:M:Y

22
0 10 1
B 3 1 C B y1 C
b(x; y) = (x1 ; x2 ) @ A@ A
1 2 y2
b(x; y) = 3x1 y1 2x2 y2 + x1 y2 + x2 y1 :

2. Soit la forme bilinéaire suivante

b(x; y) = x1 y1 + 5x2 y2 3x1 y2 3x2 y1 ;

M = [aij ]1 i;j 2 = [b(ei ; ej )]1 i;j 2 ; où B = (e1 ; e2 ) est la base canonique de R2 ;

on a 0 1
B 1 3 C
M =@ A:
3 5
En e¤et

a11 = b(e1 ; e1 ) = b((1; 0); (1; 0)) = 1;

a12 = b(e1 ; e2 ) = b((1; 0); (0; 1)) = 3;

a21 = b(e2 ; e1 ) = b((0; 1); (1; 0)) = 3;

a22 = b(e2 ; e2 ) = b((0; 1); (0; 1)) = 5:

On peut calculer la matrice de b par une autre méthode plus pratique

b(x; y) = x1 y1 + 5x2 y2 3x1 y2 3x2 y1

= x1 (y1 3y ) + x2 (5y2 3y1 )


0 2 1
B y1 3y2 C
= (x1 x2 ) @ A
3y1 + 5y2
0 10 1
B 1 3 C B y1 C
= (x1 x2 ) @ A@ A
3 5 y2
t
= XM Y;

d’où 0 1
B 1 3 C
M =@ A:
3 5

23
2.3 Changement de base
Soient B1 ; B2 deux bases de E et P la matrice de passage de B1 à B2 , si X1 et

Y1 sont les vecteurs colonnes des coordonnées de x et y respectivement dans B1 et

X2 ; Y2 sont dans B2 on a

X1 = P X2 ; Y1 = P Y2 :

Théorème 2.9 Soit b une forme bilinéaire sur E alors

MB2 (b) =t P MB1 (b) P:

Preuve.

t
b(x; y) = X1 MB1 (b) Y1
t
= (P X2 )MB1 (b) (P Y2 )
t
= X2 (t P MB1 (b) P )Y2 :

D’où

MB2 (b) = t P MB1 (b) P :

Exemple 2.10 Soit R3 munit de la base canonique B = (e1 ; e2 ; e3 ) et b une forme

bilinéaire sur R3 dé…nie par

b(x; y) = x2 y2 x1 y3 x3 y 1 :

Calculer M 0 la matrice de b dans la base B = fv1 = (1; 1; 0); v2 = (0; 1; 1); v3 = (1; 1; 1)g :

24
Solution : On a

b(x; y) = x1 ( y3 ) + x2 (y2 ) + x3 ( y1 )
0 1
B y3 C
B C
= x1 x 2 x 3 B B y 2
C
C
@ A
y1
0 10 1
B 0 0 1 C B y1 C
B CB C
= x1 x2 x3 B B 0 1 0
CB y C
CB 2 C
@ A@ A
1 0 0 y3
t
= XM Y;

d’où 0 1
B 0 0 1 C
B C
M = MB (b) = B
B 0 1 0 CC:
@ A
1 0 0
Donc on a

M 0 = MB 0 (b) =t P MB (b) P
0 10 10 1
B 1 1 0 CB 0 0 1 CB 1 0 1 C
B CB CB C
= B
B 0 1 1 CB 0 1
CB 0 C B
CB 1 1 1
C
C
@ A@ A@ A
1 1 1 1 0 0 0 1 1
0 1
B 1 0 0 C
B C
= B
B 0 1 0 C:
C
@ A
0 0 1

Remarque 2.11 Soit MB (b) la matrice associée à b dans la base B telle que

MB (b) = (aij )1 i;j n

1. Si b est symétrique alors

aij = b(ei ; ej ) = b(ej ; ei ) = aji ;

25
donc MB (b) est symétrique c.à.d

t
MB (b) = MB (b)

2. Si b est antisymétrique :

t
MB (b) = MB (b)

3. Toute forme bilinéaire (si car(K) 6= 2) sur E se décompose en la somme d’une

forme bilinéaire symétrique et une forme bilinéaire antisymétrique.

b(x; y) + b(y; x) b(x; y) b(y; x)


b(x; y) = +
2 2
= b1 (x; y) + b2 (x; y):
| {z } | {z }
2S2 (E) 2A2 (E)

Remarque 2.12 Soit b une forme bilinéaire sur un K espace vectoriel E de dimen-

sion …nie n muni d’une base B et M = matB (b): On a

M +t M M tM
M = +
2 2
= M1 + M2

il est facile de véri…er que M1 est une matrice symétrique et M2 est une matrice

antisymétrique. Ainsi M1 est la matrice de la patie symétrique b1 de b et M2 est la

matrice de la patie antisymétrique b2 de b.

2.4 Noyau et rang d’une forme bilinéaire

Dé…nition 2.13 Soit b : E E 7 ! K une forme bilinéaire symétrique ou alternée

On appelle le noyau de b l’ensemble

ker (b) = fx 2 E; 8y 2 E : b(x; y) = 0g

= fx 2 E; 8y 2 E : b(y; x) = 0g

26
Dé…nition 2.14 Le rang d’une forme bilinéaire symétrique ou alternée noté rg (b)

est le rang de sa matrice associée dans une base quelconque.

Proposition 2.15 Soient E un K-espace vectoriel de dimension …nie n, b une forme

bilinéaire symétrique ou alternée et B une base de E. Soit A = MB (b) la matrice

associée à la forme bilinéaire b dans la base B. Alors

dim ker (b) = n rg (b)

= n rg (A) :

Dé…nition 2.16 Une forme bilinéaire b sur E est dite non dégénérée (ou régulière)

si

ker b = f0g :

Elle est dite dégénérée si

ker b 6= f0g :

2.5 L’équivalence entre formes bilinéaires


Proposition 2.17 Soient E; F deux K-espaces vectoriels de dimension …nie et soient

u : E ! F un isomorphisme, b une forme bilinéaire sur F . L’application

: E E7 !K

(x; y) 7 ! b(u(x); u(y))

est une forme bilinéaire sur E.

Preuve. C’est facile à démontrer.

Dé…nition 2.18 Soit E un espace vectoriel de dimension …nie, b une forme bilinéaire

sur E .

Soit F un espace vectoriel de dimension …nie, une forme bilinéaire sur F .

On dit que b et sont équivalentes s’il existe u : E ! F un isomorphisme tel que

b(x; y) = (u(x); u(y)):

27
2.6 Orthogonalité
Soit E un K-espace vectoriel de dimension …nie et b une forme bilinéaire sur E .

Dé…nition 2.19 Soient x; y 2 E, on dit que x est b-orthogonal à y (ou orthogonal

par rapport à b) si b(x; y) = 0.On note x ?b y si pas d’ambiguïté x ? y:

Remarque 2.20 1. Si b est symétrique ou alternée la relation ?b est symétrique (

x ? y ) y ? x ).

2. Si x est b-orthogonal à y1 ; :::; yr alors x est b-orthogonal à toute combinaison

linéaire des yi .

3. Les éléments de ker (b) sont b-orthogonaux à tout élément de E.

Dé…nition 2.21 ( Base orthogonale)

Une base B = (e1 ; :::; en ) de E est dite b-orthogonale si et seulement si pour

tout i; j 2 [1; n] ; i 6= j; b(ei ; ej ) = 0: c-à-d. les vecteurs de B sont deux à deux b-

orthogonaux. 8
>
< 1; si i = j
Elle est dite b-orthonormée si et seulement si b(ei ; ej ) = ij =
>
: 0; si i 6= j

Dé…nition 2.22 (Orthogonal d’un sous-espace vectoriel) Soit b une forme bilinéaire

symétrique ou alternée sur E et F E un sous-espace vectoriel de E: L’orthogonal

de F est l’ensemble

F ? = fx 2 E; 8y 2 F; b(x; y) = 0g :

Remarque 2.23 (1) Dans la pratique, pour calculer F ? on donne successivement

comme valeurs à y les éléments d’une base de F , ce qui aboutit à un système d’équa-

tions à résoudre.

(2) Si b est une forme bilinéaire ni symétrique ni antisymétrique, on dé…nit l’or-

thonal de F par rapport à b à gauche et à droite comme suit

Fg? = fx 2 E; 8y 2 F; b(x; y) = 0g ;

Fd? = fx 2 E; 8y 2 F; b(y; x) = 0g :

28
(3) Si b une forme bilinéaire symétrique ou alternée sur E, alors ker (b) = E ? :

Dé…nition 2.24 Soient A; B E; on dit que A et B sont b-orthogonaux et on note

A ? B si et selement si

8x 2 A; 8y 2 B; b (x; y) = 0:

Proposition 2.25 Soient E un K-espace vectoriel, b une forme bilinéaire symétrique

sur E; A et B deux parties non vides de E. On note par vect(A) le sous-espace vec-

toriel de E engendré par A. On a les propriétés suivantes

1. Si A B alors B ? A? :

2. A? est un sous-espace vectoriel de E.


?
3. A? = (vect(A)) .

4. Soient A; B deux sous-espaces-vectoriels de E alors

(A + B)? = A? \ B ? :
?

5. Soit A un sous espace vectoriel de E; A A? :

Preuve. 1. Soit x un vecteur appartenant à B ? , il est donc orthogonal à tous les

vecteurs de B, et en particulier orthogonal à tous les vecteurs de A. Donc x appartient

à A? .

2. Soit A une partie non vide de E, comme 0E appartient à A? , A? est non vide.

Pour montrer que A? est un sous-espace vectoriel de E il su¢ t de montrer qu’il

est stable par combinaison linéaire.

Soient x; y 2 A? ; ; 2 K, alors pour tout vecteur z de A on a :

b( x + y; z) = b(x; z) + b(y; z)

= 0:

Car z est orthogonal à x et y. Donc x + y 2 A? et par conséquent A? est un

sous-espace vectoriel de E.

29
?
3. D’après la propriété (1), comme A vect(A), on a : (vect(A)) A? .

Réciproquement, tout élément de A? est orthogonal à tout élément de A, donc

il est othogonal à toute combinaison linéaire d’éléments de A, alors appartient à

l’orthogonal de vect(A), d’où

?
A? (vect(A)) :

On en déduit l’égalité.

4. On montre l’égalité

(A + B)? = A? \ B ?

On a

A (A + B) et B (A + B)

Donc, d’aprés la propriété 1

(A + B)? A? et (A + B)? B?;

donc

(A + B)? A? \ B ?

Réciproquement, soit x appartenant à A? \ B ? donc x est orthogonal à tout

élément de A et à tout élément de B; donc aussi à la somme d’un élément a 2 A et

d’un élément c 2 B; car

b(x; a + c) = b(x; a) + b(x; c) = 0

Par conséquent x appartient à (A + B)? .

5. Soit x appartenant à A, pour tout y appartenant à A? , on a

b(x; y) = 0:
?
? ?
Cela signi…e que x est orthogonal à tout vecteur de A , donc x appartient à A .

Cela prouve l’inclusion

30
?

A A? :

Dé…nition 2.26 On appelle vecteur isotrope tout vecteur orthogonal à lui même, c-

à-d. x 2 E est un vecteur isotrope ssi

b(x; x) = 0:

Dé…nition 2.27 Une forme bilinéaire b sur E est dite positive si pour tout x 2

E; b(x; x) 0:

Elle est dite dé…nie si pour tout x 2 E; b(x; x) = 0 ) x = 0:

Elle est dite dé…nie positive si elle est positive et dé…nie ; c.à.d. pour tout x 2

En f0g ; b(x; x) > 0:

Proposition 2.28 Une forme bilinéaire symétrique et positive est non dégénérée ssi

b(x; x) = 0 =) x = 0:

Preuve. Soit b une forme bilinéaire symétrique, positive et non dégénérée et soit

x 2 E : b(x; x) = 0; on a

8 2 K; 8y 2 E; b( x + y; x + y) 0;

d’où
2
b(x; x) + 2 b(x; y) + b(y; y) 0;

alors

2 b(x; y) + b(y; y) 0; (2.2)

on a 8y 2 E; b(y; y) 0; pour que (2.2) soit vraie pour tout 2 K et tout y 2 E; il

faut que b(x; y) = 0; donc

8y 2 E; b(x; y) = 0 =) x 2 E ? = ker (b) = f0g

=) x = 0:

31
La réciproque : Supposons que 8x 2 E; b(x; x) = 0 =) x = 0 et montrons que b est

non dégénérée
8x 2 ker (b) =) 8y 2 E; b(x; y) = 0

=) b(x; x) = 0

=) x = 0

=) ker (b) = f0g ;


d’où b est non dégénérée.

Corollaire 2.29 Toute forme bilinéaire dé…nie positive est non dégénérée.

Preuve. On a

8x 2 ker (b) =) 8y 2 E; b(x; y) = 0

=) b(x; x) = 0; pour y = x

=) x = 0; car si x 6= 0; b(x; x) > 0

=) ker (b) = f0g :

D’où b est non dégénérée.

Exercise 2.30 Dans E = R2 [X], l’espace vectoriel réel des polynômes de degré infé-

rieur ou égal à 2, on considère l’application b : E E ! R dé…nie par :


Z1
b(P; Q) = P (t):Q0 (t)dt:
0

1. Justi…er que b est une forme bilinéaire sur E.

2. Déterminer la matrice M représentant b dans la base canonique B0 = (1; X; X 2 )

de E.

3. Quel est le rang de b ?

4. b est-elle symétrique ? antisymétrique ? Déterminer la partie symétrique, b1 , et

la partie antisymétrique, b2 , de b.

5. A-t-on b(P; P ) 0 pour tout polynôme P ? à quelle condition sur P a-t-on

b(P; P ) = 0 ?

32
Solution : 1. Soient P; Q; R 2 R2 [X] et 2 R: On a

Z1
b(P + Q; R) = (P + Q) (t):R0 (t)dt
0
Z1
= [P (t) + Q(t)] :R0 (t)dt
0
Z1 Z1
0
= P (t):R (t)dt + Q(t):R0 (t)dt
0 0
= b(P; R) + b(Q; R)

et
Z1
b(P; Q + R) = P (t): (Q + R)0 (t)dt
0
Z1
= P (t) [Q0 (t) + R0 (t)] dt
0
Z1 Z1
= P (t):Q0 (t)dt + P (t):R0 (t)dt
0 0
= b(P; Q) + b(P; R)

L’application b est donc bien une forme bilinéaire sur E:

33
2. On calcule successivement

b(1; 1) = 0
Z1
b(1; X) = dX = 1
0
Z1
b(1; X 2 ) = 2XdX = 1
0
Z1
b(X; 1) = X:0dX = 0
0
Z1
1
b(X; X) = XdX =
2
0
Z1
2
b(X; X 2 ) = 2X 2 dX =
3
0
Z1
b(X 2 ; 1) = X 2 :0dX = 0
0
Z1
1
b(X 2 ; X) = X 2 dX =
3
0
Z1
1
b(X 2 ; X 2 ) = 2X 3 dX =
2
0

et donc 0 1
B 0 1 1 C
B C
M =B
B 0 1
2
2 C
3 C
@ A
0 13 21
3. On voit facilement que rg(b) = rg(M ) = 2.

4. La forme bilinéaire b n’est ni symétrique, ni antisymétrique puisque sa matrice

représentative n’est ni symétrique, ni antisymétrique. La matrice représentative de la

34
partie symétrique b1 est
0 1
B 0 1 1 C
M +t M 1B C
M1 = = B
B 1 1 1 C
C
2 2@ A
1 1 1

et celle de la partie antisymétrique b2 est


0 1
1 1
B 0 2 2 C
M t
M B C
M2 = =B
B 2
1
0 1 C:
C
2 @ 6
A
1 1
2 6
0

5. On a
Z1
b(P; P ) = P (t):P 0 (t)dt
0
1
1 2
= P (t)
2 0
1 2
= P (1) P 2 (0)
2

et donc

b(P; P ) 0 , jP (0)j jP (1)j

ce qui est le cas pour, par exemple, P (t) = 1 t: Donc on n’a pas b(P; P ) 0 pour

tout polynôme P 2 R2 [X]:

Par ailleurs, on a

b(P; P ) = 0 , P (0) = P (1)

ce qui est le cas pour, par exemple, P (t) = t (1 t). D’où la condition sur P pour

que b(P; P ) = 0 est P (0) = P (1):

35
Chapitre III

ESPACES EUCLIDIENS

Soit E un K-espace vectoriel.

3.1 Produit scalaire


Dé…nition 3.1 On appelle produit scalaire sur E toute forme bilinéaire symétrique

dé…nie positive. Il est noté par h:; :i ou h:j:i: c-à-d.

1) h:; :i forme bilinéaire symétrique.

1) 8x 2 E; hx; xi 0 avec égalité seulement pour x = 0: (8x 2 En f0g ; hx; xi > 0):

Exemple 3.2 (1) Soit E = Rn ; la forme linéaire h:; :i sur E dé…nie par : hx; yi =
Pi=n t
i=1 xi yi = X:Y est un produit scalaire appelé produit scalaire canonique (ou usuel)

de Rn :

(2) Soit E = C ([a; b] ; R) l’espace des fonctions réelles continues sur [a; b] ; la
Rb
forme linéaire h:; :i sur E dé…nie par : hf; gi = a f (t)g(t)dt est un produit scalaire:

(3) E = Mn (K) l’espace des matrices carrées, hA; Bi = T r (t A:B) est un produit

scalaire:

3.2 Espace euclidien


Dé…nition 3.3 Un espace euclidien est un espace vectoriel réel de dimension …nie

muni d’un produit scalaire.

En général tout espace vectoriel réel muni d’un produit scalaire est appelé espace

préhilbertien.

36
EUCLIDE (vers 295 avant J.-C., Grèce).

On ne sait que très peu de choses sur la vie d’Euclide en dehors du fait qu’il

ait enseigné les mathématiques à Alexandrie. Il est connu pour être le fondateur de

l’école d’Alexandrie qui in‡uença entre autres les travaux d’Archimède. Les théories

d’Euclide, dont l’œuvre maîtresse est constituée des éléments, sont une référence dans

l’histoire des mathématiques. Parmi les les cinq axiomes énoncés dans les éléments,

on trouve le célèbre postulat d’Euclide : « par tout point du plan passe une et une

seule droite parallèle à une autre droite» . Cet axiome constitue le fondement de la «

géométrie euclidienne » .

37
HILBERT, David (1862, Konigsberg - 1943, Gottingen).

Hilbert, dont l’œuvre est immense, est unanimement reconnu comme la …gure

emblématique des mathématiques du 20e siècle. Professeur à l’université de Gottingen,

il …t de cette ville le centre des mathématiques du début du 20e siècle. En août 1900,

au Second Congrès International des Mathématiciens, Hilbert a exposé 23 problèmes.

Ces problèmes ont depuis été au cœur de nombreuses recherches et il en reste à l’heure

actuelle trois qui ne sont pas résolus. Hilbert a Introduit les espaces qui portent son

nom dans le cadre de son travail sur des équations intégrales.

38
Dé…nition 3.4 (La norme) Soit E un espace préhilbertien, l’application :

k:k : E ! R
p
x 7! kxk = hx; xi

est appelée norme associée au produit scalaire.

Si E est un espace euclidien, k:k est appelée norme euclidienne.

De plus l’application

d:E E!R

(x; y) 7! kx yk

est appelée distance euclidienne.

Proposition 3.5 (Propriétées de la norme)

(1) Pour tout x 2 E; kxk 0 et kxk > 0 si x 6= 0:

(2) Pour tout x 2 E et tout 2 K; k xk = j j kxk :


x x
(3) Si x 2 En f0g ; kxk
= 1; kxk
est appelé vecteur unitaire de E:

(4) (Inégalité de Cauchy-Schwarz)

Pour tout x; y 2 E; jhx; yij kxk kyk ; avec égalité si x; y sont liés.

(5) (Inégalité de Cauchy-Schwarz dans Rn )

v v !2
u n u n
X
i=n
uX uX X
i=n X
n X
n
x i yi t x2i t yi2 ou xi y i x2i yi2 :
i=1 i=1 i=1 i=1 i=1 i=1

(6) (Inégalité de Minkovski ou inégalité triangulaire)

Pour tout x; y 2 E; kx + yk kxk + kyk ; avec égalité si x = 0 ou y = x; 2 R+ .

(7) Pour tout x; y 2 E; kx + yk2 = kxk2 + 2hx; yi + kyk2 :

(8) (Théorèm de Pythagore)

Les vecteurs x; y 2 E sont orthogonaux si et seulement si kx + yk2 = kxk2 + kyk2 :

39
PYTHAGORE (vers 500 avant J.-C., Grèce).

Mathématicien et philosophe grec, Pythagore est né dans l’île de Samos. Sa vie

et son œuvre restent mal connues. Pythagore créa à Crotone dans le sud de l’Italie

en 529 avant J .-C. une école où il professait les mathématiques et la philosophie,

attirant autour de lui un certain nombre de disciples. Ils forment une communauté

aux règles de vie très strictes dont le but est d’abord philosophique puis politique.

La philosophie pythagoricienne met en avant la toute puissance du nombre qui doit

pouvoir expliquer le monde. Les disciples doivent prêter serment de ne pas divulguer

les connaissances qui leur sont révélées. On doit aux pythagoriciens la découverte de
p p
l’incommensurabilité de 2, autrement dit le fait que 2 est un nombre irrationnel.

Pythagore fut assassiné avec de nombreux disciples dans l’incendie de l’école.

40
SCH’VARZ, Hermann-Amandus (1843, Hermsdorf - 1921, Berlin).

Schwarz …t des études de chimie et de mathématiques à Berlin. Les travaux de

Schwvarz sont marqués par une forte interaction entre l’analyse et la géométrie.

Son mémoire de thèse avait trait aux surfaces d’aire minimale. En 1884, il résolut

le problème isopérimétrique en dimension 3 qui consiste à trouver une surface d’aire

minimale qui délimite un volume maximal Schwarz réalisa également des travaux

importants en analyse fonctionnelle : étude des fonctions analytiques, des fonctions

holomorphes et de la théorie du potentiel.

41
3.3 Orthogonalité
Dé…nition 3.6 (Base orthogonale)

Soit E un espace euclidien muni d’une base B = (u1 ; :::; un ). La base B est dite

orthogonale si les vecteurs u1 ; :::; un sont deux à deux orthogonaux. i.e. hui ; uj i =

0; i 6= j: Elle est dite orthonormée si


8
>
< 1; si i = j
hui ; uj i = ij = :
>
: 0; si i 6= j

Exemple 3.7 Dans l’espace euclidien Rn muni du produit scalaire usuel hx; yi =
Pi=n n
i=1 xi yi ; la base canonique de R est orthonormée. En e¤et
P
1) Pour tout i 2 [1; n] ; hei ; ei i = ni=1 x2i = kei k2 = 1; d’où kei k = 1:

2) Pour tout i; j 2 [1; n] ; i 6= j : hei ; ej i = 0:

Proposition 3.8 Toute famille orthogonale B = fu1 ; :::; up gp n de E est libre.

Preuve. En e¤et, soient 1 ; :::; p 2 R tel que

1 u1 + ::: + p up = 0:

Soit uk 2 B; d’où on a
p
X
huk ; i ui i = 0;
i=1

alors
p
X
i huk ; ui i = 0;
i=1

mais huk ; ui i = 0; pou tout i 6= k; d’où k huk ; uk i = 0: Puisque uk 6= 0; on déduit que

k = 0; pour tout k 2 [1; p] : D’où la familles B est libre.

Théorème 3.9 (Existance d’une base orthonormée)

Tout espace euclidien admet des bases orthonormées pour son produit scalaire.

42
Théorème 3.10 (Procédé d’orthogonalisation de Gram-Schmidt)

Soit E un espace euclidien. On peut construire à partir d’une base B = (v1 ; :::; vn )

de E une base orthogonale B 0 = (u1 ; :::; un ) : La normalisation étant ensuite évidente


ui
en prenant wi = kui k
;1 i n; on obtient une base orthonormée fw1 ; :::; wn g :

Preuve. (Algorithme de Gram-Schmidt) On prend

u1 = v1
hv2 ;u1 i
u2 = v2 ku1 k2 1
u
hv3 ;u1 i hv3 ;u2 i
u3 = v3 ku1 k2 1
u ku2 k2 2
u
..
.
nP1
hvn ;uk i
un = vn kuk k2 k
u :
k=1
iP1
hvi ;uk i
Donc pour tout i 2 [2; n] ; ui = vi kuk k2 k
u : On peut véri…er facilement que
k=1
pour tout i 6= j; hui ; uj i = 0; d’où on obtient une base orthogonale B 0 = (u1 ; :::; un )
u1
pour E: Il est évident que ku1 k
; :::; kuunn k est une base orthonormée pour E:

Exemple 3.11 Soit B = fv1 = (1; 1) ; v2 = (1; 2)g une base pour R2 : Calculer une

base orthonormée pour R2 à partir de B:

Solution : D’aprés le procédé de Gram-Schmidt on a

u1 = v1 = (1; 1)
hv2 ;u1 i 1 2 3 3
u2 = v2 ku1 k2 1
u = (1; 2) 2
(1; 1) = ;
2 2
:
On remarque que hu1 ; u2 i = 0; d’où u1 = (1; 1) ; u2 = 32 ; 23 est une base or-
n o
2 0 1 1
thongonale pour R et B = u1 = 2 (1; 1) ; u2 = 2 (1; 1) est une base orthon-
p p

gonale pour R2 :

43
Proposition 3.12 Soit E un espace euclidien et F un sous-espace de E; on a

(1) F ? = fx 2 E; 8y 2 F : hx; yi = 0g
?
(2) F ? =F
?
(3) E = F F ? ( ou E = F F ?)

(4) F ? est l’unique supplémentaire de F orthogonale à F:

44
SCHMIDT, Erhard (1876, Tartu, en Estonie - 1959, Berlin).

En 1905, sous la direction de David Hilbert, Schmidt obtient Son doctorat à l’uni-

versité de Gottingen. L’activité de recherche de Schmidt a alors trait aux équations

intégrales et aux espaces dits de Hilbert. Dès 1908, il publie des articles où apparaît

le procédé d’orthogonalisation donnant ainsi naissance à ce qui s’appelle aujourd’hui

les opérateurs de Hilbert-Schmidt. En 1917, Schmidt obtint un poste de profes-

seur à l’université de Berlin. Ses travaux de recherche ont progressivement évolué

vers la topologie et il a en particulier étudié les liens entre l’aire d’une surface et Son

périmètre.

45
3.4 Matrices orthogonales
Dé…nition 3.13 On appelle matrice orthogonale toute matrice carrée réelle P d’ordre

n telle que t P P = In : C-à-d. c’est une matrice iversible et son inverse égale à sa

transposée.

Corollaire 3.14 Les vecteurs colonnes d’une matrice orthogonale forment une base

orthonormée pour le produit scalaire usuel de Rn :

Théorème 3.15 Soit E un espace euclidien muni d’une base orthonormée B: Une

nouvelle base B 0 de E est orthonormée pour ce produit scalaire ssi la matrice de

passage de B à B 0 est orthogonale.

Preuve. En e¤et si B = (e1 ; :::; en ) ; B 0 = (u1 ; :::; un ) sont deux bases orthonor-

mées de E, alors

P = PB (B 0 ) = [u1 ]B [un ]B = M at (IdE ; B 0 ; B)

u1 un
0 1
B p11 p1n C e1
B .. .. .. C ..
= B
B . . . C
C . ;
@ A
pn1 pnn en

46
d’où on a

u1 un
0 10 1
u1 B p11 pn1 CB p11 p1n C
.. B .. .. .. CB .. .. .. C
t
PP = . B . . . CB . . . C
B CB C
@ A@ A
un p1n pnn pn1 pnn
0 1
B hu1 ; u1 i hu1 ; un i C
B .. .. .. C
= B
B . . . C
C
@ A
hun ; u1 i hun ; un i
= (hui ; uj i)1 i;j n
!
X
n
= pki pkj :
k=1 1 i;j n

Puisque B 0 est orthonormée, alors pour tout i; j 2 [1; n] ; hui ; uj i = 0; si i 6= j et

hui ; ui i = 1; d’où t P P = In et P est orthogonale.

Réciproquement : Soient B; B 0 deux bases de E telle que B est orthonormée et

PB (B 0 ) est orthogonale. Montrons que B 0 est base orthonormée. On a

u1 un
0 1
B p11 p1n C e1
B .. .. .. C ..
PB (B ) = B
0
B . . . C
C .
@ A
pn1 pnn en
0 1
B p11 pn1 C u1
B .. .. .. C ..
t
P =B
B . . . C
C . ;
@ A
p1n pnn un
on a !
X
n
t
P P = In () pki pkj = In ;
k=1 1 i;j n
n
pour le produit scalaire usuel de R ; on obtient par identi…cation pour tout i; j 2

[1; n] ; hui ; uj i = 0; si i 6= j et hui ; ui i = 1: D’où B 0 est orthonormée.

47
Exemple 3.16 La matrice
0 1
B 4 7 4 C
1B C
A= B 1 4 8 C
9B
@
C
A
8 4 1

est orthogonale. En e¤et t AA = I3 ; et on pourra aussi véri…er que ces vecteurs co-

lonnes forment une base orthonormée pour R3 :

Proposition 3.17 Les seuls valeurs propres possibles dans R d’une matrice orthogo-

nale A 2 Mn (R) sont 1 et 1:

Preuve. Soit une valeur propre de A; où A est orthogonale, d’où on a

9v 2 En f0g : Av = v:

Alors,

hAv; Avi =t (Av) Av =t v t AA v =t vv = hv; vi = kvk2 ; (3.1)

d’autre part on a

hAv; Avi =t (Av) Av =t ( v) v = 2 t


vv = 2
hv; vi = 2
kvk2 : (3.2)

De (3.1) et (3.2) on obtient

2
kvk2 = kvk2 =) 2
=1

=) = 1:

3.5 Diagonalisation des matrices symétriques réelles


Si A = (aij )n est une matrice carrée d’ordre n, on dé…nit la transposée de A

en posant : t A = (aji )n : On peut voir les matrices symétriques comme les matrices

invariantes par transposition ; on dit que A est symétrique si t A = A:

48
La diagonalisation des matrices symétriques repose sur l’interprétation de la trans-

position en termes d’endomorphismes. La question est : que peut-on dire d’un endo-

morphisme dont la matrice est symétrique ? Ou encore : si f est un endomorphisme

de matrice M , que peut-on dire de l’endomorphisme dé…ni par t M ? C’est ici qu’in-

tervient le produit scalaire.

3.5.1 Adjoint d’un endomorphisme dans un espace euclidien

Soient E un espace euclidien, B = (e1 ; :::; en ) une base orthonormée, f 2 L(E) et

x; y 2 E. On pose X = matB (x) ; Y = matB (y) et A = matB (f ). On a

t
hf (x); yi = (AX)Y
t t
= X AY

= hx; f (y)i;

où f est l’endomorphisme de E dé…ni par

f (y) =t AY; d’où matB (f ) =t A:

Ainsi, le transposer de A revient à dé…nir l’endomorphisme qui permet de « commuter

f » dans hf (x); yi. Commuter signi…e ici que pour tous x; y 2 E on a

hf (x); yi = hx; f (y)i:

On notera que

1. Il n’y a qu’un seul endomorphisme f permettant de commuter f dans hf (x); yi,

2. La matrice de f dans une base orthonormée s’obtient en transposant celle de

f.

Dé…nition 3.18 Soient E un espace euclidien et f 2 L(E).

L’adjoint de f est l’endomorphisme f de E tel que

hf (x); yi = hx; f (y)i

49
pour tout x; y 2 E.

Les propriétés de la transposition des matrices se traduisent en termes d’endo-

morphismes :

Proposition 3.19 Soient E un espace euclidien, 2 R et f; g 2 L(E).

1. f =f

2. Id = Id

3. (f + g) = f + g

4. ( f ) = f

5. (f g) = g f

6. rang (f ) = rang (f )

7. det matB (f ) = det matB (f ) :

On notera que l’application linéaire

Mn (R) ! Mn (R)
t
A 7! A

se traduit par l’application linéaire

L (E) ! L (E)

f 7! f

On peut utiliser cette application pour caractériser les endomorphismes orthogonaux.

Proposition 3.20 Soient E un espace euclidien et f 2 L(E). Alors f est orthogo-


1
nal si et seulement si f = f ( où f f = IdE ):

L’endomorphisme orthogonal conserve le produit scalaire, c-à-d si pour tous x; y

de E, on a : hf (x); f (y)i = hx; yi, il est appelé aussi une isométrie.

Dé…nition 3.21 Soit E un espace euclidien. Un endomorphisme f de E est dit nor-

mal (relativement au produit scalaire de E), s’il commute avec son adjoint.

50
Il s’agit donc d’un endomorphisme f vériant l’égalité f f = f f . On peut

aussi dire que f est normal si et seulement si sa matrice dans une base orthonormée

quelconque commute avec sa transposée.

Théorème 3.22 Soient E un espace euclidien, f 2 L(E) et F un sous-espace de E:

Si F est stable par f , alors F ? est stable par f :

Preuve. On suppose que F est un sous-espace vectoriel de E stable par f , c-à-d.

f (F ) F:

Soit y 2 F ? ; pour tout x 2 F; on a f (x) 2 F , d’où

0 = hf (x); yi = hx; f (y)i:

Ainsi f (y) 2 F ? ; d’où f F? F ? ; ce qui preuve que F ? est stable par f :

Lemme 3.23 Soit f un endomorphisme quelconque d’un espace euclidien E, et soit

F un sous-espace de E. Si F est stable par f et par f , il en est de même de F ? .

Preuve. Si y est dans F ? , on a pour tout x de F ,

hf (y); xi = hy; f (x)i = 0;

car F est stable par f . On voit donc que F ? est stable par f . De la même façon, on

a:

hf (y); xi = hy; f (x)i = 0;

ce qui montre que F ? est stable par f .

Lemme 3.24 Si l’endomorphisme f de l’espace euclidien E est normal, f et f ont

les mêmes valeurs propres, et pour chaque valeur propre, ils ont le même sous-espace

propre.

51
Preuve. En e¤et, soit une valeur propre de f . Le sous-espace propre correspon-

dant E est le noyau de f Id. Il est donc stable par f et par f . Pour tous x et y

dans E , on a :

hx; f (y)i = hf (x); yi = h x; yi = hx; yi = hx; yi;

ce qui montre quef (y) = y pour tout y de E . E est donc contenu dans le sous-

espace propre de f pour la valeur propre . En permuttant les rôles de f et f , on

voit qu’il lui est égal.

3.5.2 Endomorphisme auto-adjoint

Soient E un espace euclidien et f 2 L(E).

Dé…nition 3.25 On dit que f est un endomorphisme auto-adjoint ou symétrique

ssi f = f ou hf (x); yi = hx; f (y)i:

Proposition 3.26 1) f est auto-adjoint ssi il existe une base orthonormée de E

dans laquelle sa matrice est symétrique.

2) f est auto-adjoint ssi dans toute base orthonormée de E sa matrice est sy-

métrique.

Preuve. Soit B une base orthonormée de E; f 2 L(E) et A = matB (f ) :

Si f est symétrique (auto-adjoint) on a f = f et t A = A; donc A est symétrique.

Réciproquement : Si A 2 Sn (R) matrice symétrique, en notant X = matB (x) ; Y =

matB (y), pour tout x; y 2 E

hf (x); yi =t (AX) Y =t X t
AY =t X (AY ) = hx; f (y)i;

d’où f = f , donc f est auto-adjoint.

Exercise 3.27 Soit E = Rn [X]. On pose :


Z 1
2
8 (P; Q) 2 E ; hP; Qi = 1 x2 P (x)Q(x)dx:
1

52
1. Montrer que h:; :i est un produit scalaire sur E.

2. On pose :
00
8P 2 E; (P ) = x2 1 P :

Montrer que est un endomorphisme symétrique de (E; h:; :i).

3.5.3 Diagonalisation des matrices symétriques

Alors
0 q’une 1 matrice réelle n’a pas forcement de valeurs propres réelles, par exemple

B 0 1 C
A=@ A a pour valeurs propres i et i dans C: Nous allons montrer q’une
1 0
matrice symétrique réelle ou de manière équivalente, un endomorphisme symétrique

a toute ses valeurs propres réelles et est diagonalisable dans une base orthonormée.

Lemme 3.28 Les valeurs propres d’une matrice symétrique réelle A sont toutes réelles.

Preuve. Soient une valeur propre complexe de A 2 Sn (R) Mn (C) et x un

vecteur propre associé à : On a

AX = X;

qui par conjugaison complexe donne:

AX = AX = X; A est réelle donc A = A:

D’où on a
X
n
t t
XAX = X X = : X X = t
jxi j2 ; (3.3)
i=1

car xi xi = jxi j2 où jxi j c’est le module de xi : On a aussi, du fait que A est symétrique

X
n
t t t
XAX = (AX) X = ( X) X = : X X = t
jxi j2 : (3.4)
i=1

Pn
De (3.3) et (3.4) on déduit que = ; car i=1 jxi j2 6= 0, d’où est réelle.

De lemme précédent on déduit le résultat suivant

53
Corollaire 3.29 Toute matrice symétrique réelle a n valeurs propres réelles distinctes

ou confondues.

Preuve. En e¤et, PA ( ) le polynôme caractéristique de A 2 Sn (R) est de degré

n qui admet toutes ses racines dans R:

Lemme 3.30 On suppose que n 2:

Si ; sont deux valeurs propres distinctes de A 2 Sn (R) ; alors les sous-espaces

propres E et E sont orthogonaux.

Preuve. Soit f l’endomorphisme associé à A, pour tout x 2 E et tout y 2 E ;

on a

hx; yi = h x; yi = hf (x); yi = hx; f (y)i = hx; yi = hx; yi;

d’où

( ) hx; yi = 0:

Puisque 6= ; on déduit que hx; yi = 0: D’où E et E sont orthogonaux.

Proposition 3.31 On suppose que n 2:

Si 1 est une valeur propre de f 2 S (E) (ou de A 2 Sn (R)) ; e1 un vecteur propre

associé à 1 de norme égale à 1 (ke1 k = 1) ; alors l’hyperplan H = (Re1 )? est stable

par f et la restriction de f à H est symétrique.

Preuve. H est un hyperplan puisque c’est le noyau de la forme linéaire non nulle

l:E!R

x 7! hx; e1 i

Pour tout x 2 H, on a

hf (x); e1 i = hx; f (e1 )i = hx; 1 e1 i = 1 hx; e1 i = 0;

et donc f (x) 2 H:

54
La restriction g de f à H est donc un endomorphisme de H: En designant par

(ei )2 i n une base orthonormée de H (procédé de Gram-Schmidt), B1 = (ei )1 i n est

une base orthonormée de E et la matrice de f dans cette base est


0 1
B 1 0 C
A1 = @ A;
0 B

où B 2 Mn 1 (R) est la matrice de g dans (ei )2 i n. Comme f est symétrique, il en

est de même de A1 ; donc de B et de g:

Théorème 3.32 (Spectral)

Soit A une matrice symétrique de Sn (R) : Alors, on a les propriétés équivalentes

suivantes

1: A est diagonalisable.

2: Il existe une base orthonormée de E constituée de vecteurs propres de A:


1
3: Il existe une matrice orthogonale P 2 Mn (R) ; telle que P AP =t P AP = D

soit diagonale.

En termes d’endomorphismes on a le théorème spectral suivant

Théorème 3.33 Soit f 2 S(E) un endomorphisme symétrique: Alors, on a les pro-

priétés équivalentes suivantes

1: f est diagonalisable.

2: Il existe une base orthonormée de E constituée de vecteurs propres de f:

Exemple 3.34 Diagonaliser la matrice suivante dans une base orthonormée.


0 1
B 1 1 1 C
B C
A=B B 1 1 1 C:
C
@ A
1 1 1

55
Solution : Le polynôme caractérestique de A est

PA ( ) = (1 ) ( + 2)2 ;

donc les valeurs propres de A sont 1 = 1 v.p.s.; 2 = 2 v.p.d. et les vecteurs propres

associés sont : v1 = (1; 1; 1) pour 1 et v2 = ( 1; 1; 0) ; v3 = ( 1; 0; 1) pour 2: On

remarque que hv1 ; v2 i = hv1 ; v3 i = 0 mais hv2 ; v3 i = 1 6= 0:

En utilisant le procédé de Gram-Schmidt pour fv2 ; v3 g on obtien deux vecteurs

orthogonaux :

u2 = v2
hv3 ; u2 i 1
u3 = v3 u2 = (1; 1; 2) ;
ku2 k2 2

si on pose u1 = v1 ; on obtient une base orthogonale B = fu1 ; u2 ; u3 g pour R3 : D’où

une base orthonormée

1 1 1
w1 = p (1; 1; 1) ; w2 = p ( 1; 1; 0) ; w3 = p (1; 1; 2)
3 2 6
pour R3 : D’où A est diagonalisable telle que
0 1
B 1 0 0 C
B C
t
P AP = D = B B 0 2 0 CC;
@ A
0 0 2

où 0 1
p p
B 2 3 1 C
1 B p p C
P =p B 2 3 1 C
6B
@ p
C
A
2 0 2
est une matrice orthogonale.

Exercise 3.35 Soit 0 1


1 1 1
B 2 4 4 C
B C
M =B
B
1
4
1
3
5
12
C
C
@ A
1 5 1
4 12 3

56
1. Prouver que la suite de matrices (M n ) converge.

2. Soit N = lim M n : Caractériser géométriquement l’endomorphisme associé à


n!1

N. 0 1
B u0 C
B C
3. Soit (Xn ) la suite de vecteurs de R3 dé…nie par X0 = B
B v0
C et Xn+1 = M Xn .
C
@ A
w0
Prouver que la suite (Xn ) converge et déterminer sa limite en fonction de u0 ; v0 et

w0 .

Solution :

1. La matrice M étant symétrique réelle, elle est diagonalisable dans une base
1
orthonormée. Ses valeurs propres sont 1; 14 et 12
. On cherche ensuite une base de

vecteurs propres et on trouve

E1 = h(1; 1; 1)i

E 1 = h( 2; 1; 1)i
4

E 1 = h(0; 1; 1)i
12

1 1 1
B0 = p (1; 1; 1) ; p ( 2; 1; 1) ; p (0; 1; 1)
3 6 2
est une base orthonormée pour R3 : Donc il existe une matrice othogonale P telle que
0 1
p
B 2 2 0 C
1 B p p C
P =p B B 2 1 3 C
C
6@
p p A
2 1 3

et 0 1
B 1 0 0 C
B C
P 1 M P =t P M P = D = B 1
B 0 4 0 C:
C
@ A
1
0 0 12

57
1
Puisque M = P DP alors
0 1
B 1 0 0 C
B C
n
M = PD P n 1
=PB
B 0
1 n
4
0 CP
C
1
;
@ A
1 n
0 0 12

où D est la matrice diagonale précédente, on constate sans peine que M n tend vers
0 1
B 1 0 0 C
B C 1
N = PB B 0 0 0 CP
C
@ A
0 0 0
0 1
B 1 1 1 C
1B
B 1 1 1 C
C
= B C
3@ A
1 1 1

2. Notons (u; v; w) les 3 vecteurs colonnes de P (i.e une base de vecteurs propres

de N ). Alors si f est l’endomorphisme associé à N , on a f (u) = u; f (v) = 0 et

f (w) = 0 (car les valeurs propres de N sont 1 et 0) f est donc la projection sur

vect(u) parallèlement à vect(v; w). Mais puisque la famille (u; v; w) est une famille

orthonormée (on a diagonalisé une matrice symétrique), f est en réalité la projection

orthogonale sur vect(u) = ker(M In ) = E =1 .

3. Par une récurrence simple, on a pour tout n 2 N;

Xn = M n X0 = M n = P D n P 1
X0 :

Passans à la limite, on trouve que (Xn ) converge vers le vecteur


0 1
B u0 + v0 + w0 C
1B C
Y = N X0 = B u + v + w C:
3B @
0 0 0 C
A
u0 + v0 + w0

58
Chapitre IV

FORMES QUADRATIQUES

Dans tout ce chapitre K désigne un corps commutatif de caractéristique di¤érente

de deux, ce qui signi…e que dans ce corps 1 + 1 6= 0. Moralement cela revient à dire

que dans K on peut diviser par deux. E un K-espace vectoriel de dimension …nie.

On s’intéresse ici aux fonctions de K n dé…nies par des formules du type

2 2
q(x1 ; :::; xn ) = 1 x1 + ::: + n xn + 12 x1 x2 + ::: + n 1;n xn 1 xn

où les i et les ij sont des scalaires …xés. Ce sont les fonctions dé…nies par les

polynomes homogènes de degré deux. On les appelle formes quadratiques. Elles ap-

paraissent par exemple dans l’étude des courbes algébriques de degré deux. Dans R2 ,

une courbe de degré deux est une partie C dé…nie par une équation

ax2 + by 2 + cxy + dx + ey + f = 0;

où (a; b; c; d; e; f ) 2 R6 et (a; b; c) 6= (0; 0; 0). Le membre de gauche de cette équation

est constitué d’une forme quadratique (ax2 +by 2 +cxy), d’une forme linéaire (dx+ey)

et d’une constante (f ). L’étude des formes quadratiques permet de montrer assez

facilement que C est une conique.

4.1 Formes quadratiques


Toute forme bilinéaire b sur E dé…nit une fonction q sur E par

q(x) = b(x; x)

Une telle fonction est appelée forme quadratique sur E. Il se trouve qu’il est inutile de

faire appel à toutes les formes bilinéaires pour obtenir toutes les formes quadratiques

sur E. En e¤et,

59
- premièrement, si b est alternée alors pour tout x 2 E; b(x; x) = 0,

- et deuxièmement, b(x; x) = bsym (x; x) + balt (x; x) = bsym (x; x).

Ainsi, b dé…nit la même forme quadratique que la forme symétrique bsym : deux

formes bilinéaires qui di¤érent d’une forme alternée dé…nissent la même forme qua-

dratique. Nous adopterons la

Dé…nition 4.1 Soit E un K-espace vectoriel et q : E ! K. On dit que q est une

forme quadratique sur E s’il existe une forme bilinéaire symétrique b sur E telle que

pour tout x 2 E;

q(x) = b(x; x)

On dit que q est la forme quadratique associée à b et b la forme polaire de q. On note

Q(E) l’ensemble des formes quadratiques sur E:

Nous avons une surjection naturelle

S2 (E) ! Q(E)

et aussi

L2 (E) ! Q(E):

Exemple 4.2 (1) La fonction

R ! R

x 7! x2

est la forme quadratique sur R associée à la forme bilinéaire

R R ! R

(x; y) 7! xy:

(2) La fonction

R2 ! R

x 7! x21 + 5x22 6x1 x2

60
est la forme quadratique sur R2 associée à la forme bilinéaire

R2 R2 ! R

(x; y) 7! x1 y1 + 5x2 y2 3x1 y2 3x2 y1 :

(3) La fonction

C ([0; 1] ; R) ! R
Z 1
f 7! f 2 (t) dt
0

est la forme quadratique sur C ([0; 1] ; R) associée à la forme bilinéaire

C ([0; 1] ; R) C ([0; 1] ; R) ! R
Z 1
(f; g) 7! f (t) g(t)dt:
0

(4) La fonction

Mn (K) ! R

M 7! T r M 2

est la forme quadratique sur Mn (K) associée à la forme bilinéaire

Mn (K) Mn (K) ! R

(M; N ) 7! T r (M N ) :

Proposition 4.3 (Identités remarquables)

Soit b une forme bilinéaire symétrique sur E; q la forme quadratique associée à b;

on a pour tout x; y 2 E
2
1) q( x) = q(x); 8 2 K

2) q(x + y) = q(x) + q(y) + 2b(x; y);

3) b(x; y) = 21 [q(x + y) q(x) q(y)]

61
Preuve. 1) On a

q( x) = b( x; x) = b(x; x)
2 2
= b(x; x) = q(x)

2)

q(x + y) = b(x + y; x + y)

= b(x; x) + b(y; y) + b(x; y) + b(y; x)

= q(x) + q(y) + 2b(x; y) (car b est symétrique)

3) A partir de 2) on a

1
2b(x; y) = q(x + y) q(x) q(y) , b(x; y) = [q(x + y) q(x) q(y)]:
2

Proposition 4.4 Si q est un forme quadratique sur E, alors il existe une unique

forme bilinéaire symétique sur E associée à q qu’on appelle forme polaire de q dé…nie

par
1
b(x; y) = [q(x + y) q(x) q(y)]:
2

Preuve. La forme quadratique q est dé…nie par

q(x) = b0 (x; x); x 2 E;

où b0 est une forme bilinéaire sur E. L’application b dé…nie sur E E par

1
b(x; y) = (b0 (x; y) + b0 (y; x))
2

est bilinéaire et symétrique avec

b(x; x) = q(x)

62
pour tout x 2 E, ce qui prouve l’existence de b. Comme b est bilinéaire symétrique,

on a pour tout x; y 2 E
1
b(x; y) = (q(x + y) q(x) q(y)) (d’aprés la proposition 4.3)
2
d’où b existe et elle est unique.

Exemple 4.5 Soit la forme quadratique q dé…nie par :

q : R2 ! R

q(x) = 2x21 + 5x22 4x1 x2 ;

tel que x = (x1 ; x2 ):Sa forme polaire est


1
b(x; y) = [q(x + y) q(x) q(y)]
2
= x1 y1 + 5x2 y2 2x1 y2 2x2 y1

Remarque 4.6 Quand la forme quadratique est donée par un polynôme homogène de

degré deux, la forme polaire b s’obtient en polarisant chaque monôme de ce polynôme.

Un monôme de la forme (aii x2i ) est polarisé en (aii xi yi ) et un monôme de la forme


aij
(aij xi xj ) est polarisé en 2
(xi yj + xj yi ).

Exemple 4.7 1) Dans l’exemple 4.5 on a


4
b(x; y) = 2x1 y1 + 5x2 y2 (x1 y2 + x2 y1 )
2
= 2x1 y1 + 5x2 y2 2x1 y2 2x2 y1

2) La forme quadratique

q : R3 ! R

q(x) = 7x21 + 6x1 x2 + 5x2 x3 ;

se polairise en

b : R3 R3 ! R
5 5
b(x; y) = 7x1 y1 + 3x1 y2 + 3x2 y1 + x2 y3 + x3 y2 :
2 2

63
4.2 Matrice d’une forme quadratique
Soient E un espace vectoriel de dimension n, x et y des éléments de E, B = (ei )

une base de E et b une forme bilinéaire sur E. D’aprés le chapitre 2 on a montrer que

b(x; y) =t XM Y; où

M = MB (b) = (aij )1 i;j n = (b(ei ; ej ))1 i;j n

Dé…nition 4.8 Soient E un espace vectoriel de dimension n, (ei ) une base de E; b

une forme bilinéaire symétrique sur E et q une forme quadratique associée à b: La

matrice MB (b) est aussi appelée matrice de q dans B; c-à-d. MB (q) = MB (b) ; d’où

on a pour tout x 2 E

q(x) = b(x; x) =t XM X;

où X est la matrice colonne des coordonnées de x: On déduit que


X
n
q(x) = aij xi xj
i;j=1
X n X
n
= aii x2i +2 aij xi xj :
i=1 1 i<j 1

Si on écrit l’expression précédente sous la forme


X
n X
n
q(x) = cii x2i + cij xi xj ; où cii = aii et cij = 2aij
i=1 1 i<j 1

on a 0 1
c12 c1n
c
B 11 2 2 C
B c ... .. C
B 12 c22 . C
B 2 C
MB (q) = B . C
B . ... ..
. cn 1;n C
B . 2 C
@ A
c1n cn 1;n
2 2
cnn
qui est une matrice symétrique.

Exemple 4.9 Soit la forme quadratique

q : R3 ! R; où q(x) = x21 + 7x22 + 6x1 x2 2x1 x3 + 3x2 x3 ;

64
on a 0 1
B 1 3 1 C
B C
M =B
B 3 7
3 C:
2 C
@ A
1 23 0

Soit q une forme quadratique donnée et b sa forme polaire, on a les résultats

suivants

Remarque 4.10 (1) Si q est une forme quadratique donnée par


X
n X
n
q(x) = aii x2i +2 aij xi xj
i=1 1 i<j 1

alors sa forme polaire est donnée par


X
n X
n
b(x; y) = aii xi yi + aij (xi yj + xj yi ) :
i=1 1 i<j 1

(2) Une forme quadratique q est dite non dégénérée quand sa forme polaire b l’est.

(3) On dé…nit le noyau et le rang d’une forme quadratique comme ceux de sa forme

polaire. C-à-d. ker (q) = ker (b) ; rang (q) = rang (b) :

(4) On dit que x 2 E est isotope par rapport à q si q(x) = 0:

(5) L’orthogonal d’un sous-espace vectoriel de E par rapport à q est son orthogonal

par rapport à b:

Exemple 4.11 Soit la forme quadratique : q : R2 ! R dé…nie par

q(x) = x21 x22 :

La matrice de q dans la base canonique de R2 est


0 1
B 1 0 C
M =@ A
0 1

La forme polaire de q est

b(x; y) = x1 y1 x2 y2

65
Le noyau de q est

ker (q) = x 2 R2 : b(x; y) = 0; 8y 2 R2


8 8
>
< b(x; e1 ) = 0 >
< x1 = 0
,
>
: b(x; e2 ) = 0 >
: x2 = 0
8
>
< x1 = 0
,
>
: x2 = 0

d’où

ker(q) = f0R2 g

On déduit que q est non dégénérée et rang(q) = 2:

Les vecteurs isotropes de q sont les deux droites vectorielles d’équations : x2 =

x1 et x2 = x1 :

4.3 Réduction des formes quadratiques


4.3.1 Réduction dans le cas général

On rappelle qu’une base B = (e1 ; :::; en ) d’un K-espace vectoriel E est dite b-

orthogonale où b est une forme bilinéaire sur E si et seulement si pour tout i; j 2

[1; n] ; i 6= j; b(ei ; ej ) = 0; c-à-d. les vecteurs de B sont deux à deux b-orthogonaux.

On dit que B = (ei ) est b-orthonormée si pour tout (i; j) 2 f1; :::; ng2 ,
8
>
< 1 si i = j
b(ei ; ej ) = ij = ;
>
: 0 si i 6= j

où ij est le symbole de Kronecker.

Nous savons que tout espace euclidien E possède des bases orthonrmées et que

dans une telle base, b(x; y) prend la forme


X
n X
n
b(x; y) = xi yj et q(x) = x2i :
i=1 i=1

Nous allons généraliser ce résultat aux formes quadratiques.

66
Proposition 4.12 On notera que les assertions suivantes sont équivalentes

1. La base B = (ei ) est q-orthogonale.


Pn 2
2. Dans la base B = (ei ) la forme q s’écrit sous la forme q(x) = i=1 i xi :

3. Dans la base B = (ei ) la matrice de q est diagonale.

De la même manière nous avons les équivalences suivantes

1. La base B = (ei ) est q-orthonormée.


Pn
2. Dans la base B = (ei ) la forme q s’écrit sous la forme q(x) = i=1 x2i .

3. Dans la base B = (ei ) la matrice de q est la matrice identité.

Dé…nition 4.13 On appelle réduction d’une forme quadratique q, la recherche d’une

base orthogonale de E, tel que sur cette base q s’écrit sous la forme

X
n
2
q(x) = i xi ; i 2R
i=1

et les (xi ) sont les coordonnées de x dans cette base. De plus la matrice de q dans

cette base est diagonale.

Théorème 4.14 Nous donnons deux formulations di¤érentes du même résultat.

1. Tout espace pseudo-euclidien (E; q) de dimension …nie possède une base ortho-

gonale.

2. Tout espace pseudo-euclidien (E; q) de dimension …nie possède une base dans

laquelle la matrice de q est diagonale.

Autrement dit ils existent r 2 f1; :::; ng, une base (e1 ; :::; en ) de E et des scalaires

non nuls 1 ; :::; r tels que

67
0 1
..
B 1 0 . C
B .. ... .. C
B . . C
B C
B C
B .. C
B . r 0 C
M (q) = B
B ..
C;
C
B 0 0 . C
B C
B .. .. .. C
B . . . C
B C
@ A
..
. 0 0
évidemment r est le rang de q.

Preuve. Nous procédons par récurrence sur n, la dimension de E. Pour n = 1 le

résultat est trivial.

Supposons le vrai pour les espaces de dimension n 1. Soit (E; q) un espace de

dimension n, si q = 0, le résultat est trivial, nous supposons donc qu’il existe a 2 E

tel que q(a) 6= 0.

On pose

hai? = fx 2 E; b(x; a) = 0g:

On l’appelle l’orthogonal de a; c’est le sous-espace des éléments q-orthogonaux à a.

On notera que

hai? = ker b( ; a):

Et puisque la forme linéaire b( ; a) n’est pas nulle, l’orthogonal de a est un hyperplan

= hai? , nous avons


de E. Puisque par ailleurs a 2

E = hai hai?

D’après l’hypothèse de récurrence, l’orthogonal de a possède une base q-orthogonal

(e1 ; :::; en 1 ): La famille (a; e1 ; :::; en 1 ) est alors une base q-orthogonal de E.

Nous souhaiterons faire une remarque sur le cas réel. Nous disposons en e¤et dans

ce cas d’un raccourci : le théorème spectral. Ce dernier a¢ rme que toute matrice

symétrique réelle d’ordre n est diagonalisable dans une base orthonormée de l’espace

68
euclidien Rn ainsi, si A désigne la matrice de q dans une base de E, la symétrie de

cette matrice implique l’existence d’une matrice orthogonale P 2 Mn (R) telle que
1
P AP soit diagonale. Or le fait que p soit orthogonale implique que

t 1
P =P

et dont t P AP est diagonale, d’où le résultat. Cette preuve algébrique établit un

curieux lien entre les formes quadratiques et les endomorphismes. On peut dans la

pratique pro…ter de ce lien pour réduire notre forme réelle q. Concrètement, on appelle

(e) la base de E dans laquelle la matrice de q est A et on diagonalise cette dernière.

Cela consiste à chercher les valeurs propres 1 ; :::; n de A, et des vecteurs propres

v1 ; :::; vn 2 Rn associées a chacune de ces valeurs. On prend soin de choisir ces vecteurs

de manière à ce qu’ils forment une base orthonormée de Rn . Ce dernier point est

fondamental, on obtient ainsi une matrice diagonale D

1
D=P AP

où P est la matrice de la base (v1 ; :::; vn ) dans la base canonique, ensuite on transpose

cette écriture à (E; q) en dé…nissant la base (a1 ; :::; an ) de E par

M(e) (a1 ; :::; an ) = P:

Dans cette base q est réduite.

Remarque 4.15 Le théorème spectral est un résultat propre au corps R. Première-

ment, dans un espace complexe E il n’y a pas de bases orthonormées : pour obtenir

une notion analogue il faut remplacer la notion de produit scalaire par la notion de

produit hermitien. Deuxièmement, il existe des matrices symétriques complexes qui

ne sont pas diagonalisables, c’est le cas par exemple de


0 1
B 1 i C
@ A
i 1

On laisse au lecteur le soin de le véri…er.

69
4.3.2 Réduction en carrés de Gauss

Nous donnons ici une autre manière d’établir le théorème de réduction. Cette

méthode dûe à Carl Friedrich Gauss est pratique pour les exercices, et permet

d’obtenir explicitement et facilement une base orthogonale. Commençons par refor-

muler le théorème de réduction.

Soit q une forme quadratique non nulle sur un K-espace vectoriel E de dimension

…nie n muni d’une base B = (e1 ; e2 ; :::; en ) de sorte que, pour tout x = x1 e1 + ::: +

xn en 2 E.

X
n X
q(x) = aii x2i + 2aij xi xj : (4.1)
i=1 1 i<j n

La méthode de Gauss consiste à écrire q sous la forme d’une combinaison de carrés de

formes linéaires indépendantes l1 ; :::; ln en procédent par récurrence sur n: Distingons

deux cas

Premier cas : L’un des aii est non nul, par exemple a11 6= 0: Alors on sépare

dans (4.1) les monomes contenant x1 des autres

q(x) = a11 x21 + x1 f (x2 ; :::; xn ) + g (x2 ; :::; xn ) ;

où f est une forme linéaire et g est une forme quadratique en les x2 ; :::; xn : On écrit

alors
2
f (x2 ; :::; xn ) (f (x2 ; :::; xn ))2
q(x) = a11 x1 + + g (x2 ; :::; xn )
2a11 4a11
= a11 (l1 (x))2 + q 0 (x);

f (x2 ;:::;xn ) (f (x2 ;:::;xn ))2


où l1 (x) = x1 + 2a11
une forme linéaire et q 0 (x) = 4a11
+ g (x2 ; :::; xn )

une forme quadratique en les x2 ; :::; xn : On applique le même procédé de récurrence

sur q 0 (x) on obtient des formes linéaires indépendantes de la forme linéaire l1 telles

que
X
r
q(x) = i (li (x))2 ; r n:
i=1

70
Deuxième cas : Tous les aii sont nuls. Dans ce cas q(x) ne s’écrit qu’avec des

rectangles
X
q(x) = 2aij xi xj :
1 i<j n

Il existe i < j tel que aij est non nul et quitte à permuter les indices nous pouvons

supposer que a12 6= 0: Décomposons q(x) selon les termes qui contiennent x1 , ceux

qui contiennent x2 et les autres :

q(x) = a12 x1 x2 + x1 f (x3 ; :::; xn ) + x2 g (x3 ; :::; xn ) + h (x3 ; :::; xn ) ;

où f; g sont des formes linéaires et h est une forme quadratique en les x3 ; :::; xn : On

écrit alors

f g
q(x) = a12 x1 x2 + x1 + x2 +h
a12 a12
g f fg
= a12 x1 + x2 + + h:
a12 a12 a12

On utilise l’identité remarquable

1
ab = (a + b)2 (a b)2 ;
4

on obtient
2 2
a12 f g a12 g f fg
q(x) = x1 + x2 + + x1 x2 + +h
4 a12 a12 4 a12 a12 a12
a12 a12
= (l1 (x))2 (l2 (x))2 + q 0 (x);
4 4

où l1 ; l2 sont deux formes linéaires indépendantes et q 0 (x) est une forme quadratique

en les x3 ; :::; xn : On itère le même procédé sur la forme quadratique q 0 (x) on obtient

X
r
q(x) = i (li (x))2 ; r n;
i=1

où les l1 ; :::; lr sont des formes linéaires indépendantes et l’entier r n est le rang de

q:

71
Dé…nition 4.16 (Signature d’une forme quadratique) Si K = R; il existe une

base orthogonale B = (ei )1 i n où la matrice de q est diagonale. Soit s le nombre de

coe…cients strictement positifs et soit t le nombre de coe…cients strictement négatifs.

Le couple (s; t) s’appelle la signature de q noté sgn (q) : C-à-d.

s = card fi 2 [1; n] : q(ei ) > 0g ; t = card fi 2 [1; n] : q(ei ) < 0g ;

et r = s + t est le rang de q.

Remarque 4.17 1. Si t = 0, on dit que la forme q est positive.

2. Si s + t = n, on dit que la forme est nondégénérée.

3. Si s = n, on dit que la forme est dé…nie positive. Dans ce cas, la forme polaire

b est un produit scalaire et l’espace muni de b est euclidien.

Théorème 4.18 (Loi d’inertie de Sylvester) La signature (s; t) est un invariant

de q, c-à-d. la signature de q ne dépend pas du choix de la base q-orthogonale. Autre-

ment dit :

Soit q une forme quadratique de rang r sur un espace vectoriel réel E de dimension

…nie. Alors, il existe (e1 ; :::; en ) une base de E, et des entiers s et t tels que, pour tout

vecteur x = x1 e1 + ::: + xn en de E; on ait

q(x) = x21 + ::: + x2s x2s+1 ::: x2s+t ; s + t = r:

Le couple (s; t) est unique.

Remarque 4.19 La loi d’inertie de Sylvester est un théorème de classi…cation des

formes quadratiques sur un espace vectoriel réel de dimension …nie.

Sylvester applique ces résultats à la mécanique et analyse l’énergie à transmettre

à un solide pour lui donner une vitesse de rotation. C’est ce qu’on désigne par principe

d’inertie de Sylvester .

72
Exemple 4.20 Soit la forme quadratique dé…nie sur R3 par

q(x) = x21 + 3x22 4x23 + 6x1 x2 + 8x1 x3 :

On a

q(x) = x21 + 3x22 4x23 + 6x1 x2 + 8x1 x3

= x21 + x1 (6x2 + 8x3 ) + 3x22 4x23

= (x1 + 3x2 + 4x3 )2 (3x2 + 4x3 )2 + 3x22 4x23

= (x1 + 3x2 + 4x3 )2 6x22 20x23 24x2 x3

= (x1 + 3x2 + 4x3 )2 6 x22 + 4x2 x3 20x23

= (x1 + 3x2 + 4x3 )2 6 (x2 + 2x3 )2 + 4x23

La signature de q est sgn(q) = (2; 1), et rang(q) = 3; ceci implique que

dim ker q = f0g :

D’où cette forme est non dégénérée.

Recherche d’une base orthogonale : On pose


8
>
> 0
x1 = x1 + 3x2 + 4x3
>
>
<
0
x2 = x2 + 2x3 ;
>
>
>
>
: x 0 = x3
3

alors 8
>
> 0
x1 = x1
0
3x2 + 2x3
0
>
>
<
0 0

> x2 = x2 2x3
>
>
>
: x3 = x0
3
0
D’où X = P X ; où 0 1
B 1 3 2 C
B C
P =B
B 0 1 2 C
C
@ A
0 0 1

73
est la matrice de passage de la base canonique à la base orthogonale.

0
B = fu1 = (1; 0; 0) ; u2 = ( 3; 1; 0) ; u3 = (2; 2; 1)g :

0
L’expression de q dans B est

q(x) = x02
1 6x02 02
2 + 4x3

0 0 0
tel que x1 ; x2 ; x3 sont les coordonnées de x dans B 0 .
0
La matrice de q dans B est
0 1
B 1 0 0 C
B C
D=B
B 0 6 0 C
C:
@ A
0 0 4

Exemple 4.21 Soit la forme quadratique dé…nie sur R4 par

q(x) = x1 x2 + 2x1 x3 + 2x1 x4 + x2 x3 + 4x2 x4 + 2x3 x4 :

Réduction en carrés de Gauss :

q(x) = (x1 x2 + 2x1 x3 + 2x1 x4 + x2 x3 + 4x2 x4 ) + 2x3 x4

= [x1 x2 + x1 (2x3 + 2x4 ) + x2 (x3 + 4x4 )] + 2x3 x4

= (x1 + (x3 + 4x4 )) (x2 + (2x3 + 2x4 )) (x3 + 4x4 ) (2x3 + 2x4 ) + 2x3 x4
1 1
= (x1 + x2 + 3x3 + 6x4 )2 (x1 x2 x3 + 2x4 )2 2 x23 + 4x24 + 4x3 x4
4 4
1 1
= (x1 + x2 + 3x3 + 6x4 )2 (x1 x2 x3 + 2x4 )2 2 (x3 + 2x4 )2
4 4
1
2
1
2 hp i2
= (x1 + x2 + 3x3 + 6x4 ) (x1 x2 x3 + 2x4 ) 2 (x3 + 2x4 ) :
2 2

On peut déduire que sgn(q) = (1; 2) ; rang(q) = 3; dim ker(q) = 1; d’où q est dégéné-

rée.

74
La recherche d’une base orthogonale : Soit le changement de variables
8
>
> x01 = 12 (x1 + x2 + 3x3 + 6x4 )
>
>
>
>
>
< x0 = 1 (x1 x2 x3 + 2x4 )
2 2
> p
>
> 0
x3 = 2 (x3 + 2x4 )
>
>
>
>
: x0 = x
4 4

D’où 8
>
> x1 = x01 + x02 p1 x0 2x04
>
> 2 3
>
> p
>
< x2 = x0 x0
1 2 2x03 + 2x04
>
>
>
> x3 = p12 x03 2x04
>
>
>
: x = x0
4 4
0
D’où X = P X ; 0 1
1 1 p1 2
B 2 C
B p C
B 1 1 2 2 C
B C
P =B C;
B p1
C
B 0 0 2
2 C
@ A
0 0 0 1
où P est la matrice de passage de la base canonique à la base orthogonale B 0

1 p 1
B0 = v1 = (1; 1; 0; 0) ; v2 = (1; 1; 0; 0) ; v3 = p ; 2; p ; 0 ; v4 = ( 2; 2; 2; 1) :
2 2
0
L’expression de q dans B est

q(x) = x02
1 x02
2 x02
3

0
La matrice de q dans B est
0 1
1 0 0 0
B C
B C
B 0 1 0 0 C
B C
D=B C:
B C
B 0 0 1 0 C
@ A
0 0 0 0

75
4.3.3 La réduction dans une base de vecteurs propres

Théorème 4.22 Soit q une forme quadratique dé…nie sur l’espace vectoriel réel E

muni d’une base B: Notons M = matB (q); alors il existe au moins une base q orthogonale

sur laquelle q s’écrit sous la forme :

02 02 02
q(x) = 1 x1 + 2 x2 + ::: + n xn ;

0
où les xi 1 i n
sont les coordonnées de x dans cette base, les i 2 R sont les valeurs

propres de M:

Preuve. Soit B = fe1 ; :::; en g une base de E: Puisque M est une matrice symé-

trique réelle, alors il existe une matrice orthognale P (t P P = I) telle que :

1 t
D = P
0 MP = P:M:P
1
B n ::: 0 C
B . . .. C
= B
B .
. .. . C
C soit diagonale
@ A
0 ::: n

Alors fP e1 ; :::; P en g est une base q orthogonale. Nous pouvons alors écrire

t
q(x) = X:M:X
t
q(x) = X: P:D:t P :X
t
q(x) = X:P :D: t P:X
t t
q(x) = P:X D: t P:X ;

on a X = P X 0 ; donc X 0 = P 1
X =t P X d’où
X
n
02
t 0 0
q(x) = X :M:X = i xi ;
i=1

où les i sont les valeurs propres de M .

Exemple 4.23 Soit q : R2 ! R ,

q(x; y) = x2 + 11y 2 16xy

76
B est la base canonique de R2 : On a
0 1
B 1 8 C
M = MB (q) = @ A matrice symétrique
8 11

- Calcul des valeurs propre de la matrice M :

pM ( ) = det(M I) = ( + 5)( 15)

Les valeurs propres sont 1 = 15; 2 = 5.

- Les sous espaces vectoriels associées aux valeurs propres sont :

E 1 =< (1; 2) >; E 2 =< (2; 1) >

On choisit une base q-orthogonale, et orthonormée par rapport au produit scalaire qui

est
1 1
B0 = e01 = p (1; 2) ; e02 = p (2; 1)
5 5
0 1
1 B 1 2 C
P =p @ A matrice orthogonale.
5 2 1

M 0 = M (q; B 0 ) =t P M P
0 10 1 0 1
1 B 1 2 CB 1 8 C 1 B 1 2 C
= p @ A@ Ap @ A
5 2 1 8 11 5 2 1
0 1 0 1
B 15 0 C B 1 0 C
= @ A=@ A:
0 5 0 2

L’expression de q dans B 0 est :

0
q(x) =t X 0 M 0 X 0 = 15x02 5y 2 :

Exemple 4.24 Soit q : R3 ! R

q(x) = x21 x22 x23 + 2 (x1 x2 + x1 x3 + x2 x3 )

77
B est la base canonique de R3 : On a
0 1
B 1 1 1 C
B C
M = MB (q) = B B 1 1 1 CC matrice symétrique
@ A
1 1 1

- Calcul des valeurs propre de la matrice M

pM ( ) = det(M I) = (1 )( + 2)2 :

Les valeurs propres sont 1 = 1; 2 = 2.

- Les sous-espaces propres sont

E 1 =< v1 = (1; 1; 1) >; E 2 =< v2 = ( 1; 1; 0) ; v3 = ( 1; 0; 1) >

On remarque que < v1 ; v2 >=< v1 ; v3 >= 0; < v2 ; v3 >= 1 6= 0; en utilisant le procédé

de Gram-Schmidt on obtient
8
>
> u1 = v1
>
>
<
> u2 = v2
>
>
>
: u3 = v3 <v3 ;v2 > 1
kv2 k2
:v2 = 2
(1; 1; 2)

On peut choisir une base q orthogonale formée de vecteurd propres

1
(1; 1; 1) ; ( 1; 1; 0) ; (1; 1; 2) ;
2

et une base orthonormée par rapport au produit scalaire canonique de R3 formée de

vecteurd propres

1 1 1
B0 = p (1; 1; 1) ; p ( 1; 1; 0) ; p (1; 1; 2)
3 2 6

et on a 0 1
B 1 0 0 C
B C
M 0 = matB 0 (q) =t P M P = B
B 0 2 0 CC;
@ A
0 0 2

78
où 0 1
p p
B 2 3 1 C
1 B p p C
P =p B 2 3 1 C
C matrice orthogonale
6B
@ p A
2 0 2
L’expression de q dans B 0

q(x) =t X 0 M 0 X 0 = x02
1 2x02
2 2x02
3

où x01 ; x02 ; x03 sont les coordonnées de x dans B 0 : De plus on a


0 10 1
p p p
B 2 2 2 C B x1 C
1 B p p CB C
X 0 = tP X = p B 3 3 0 CB x C
6B@
CB 2
A@
C
A
1 1 2 x3
0 1
p
B 2 (x1 + x2 + x3 ) C
1 B p C
= p B B 3 (x1 x2 ) C C:
6@ A
(x1 + x2 2x3 )

D’où

q(x) = x02
1 2x02
2 2x02
3
1 1
= (x1 + x2 + x3 )3 (x1 x2 )2 (x1 + x2 2x3 )2 :
3 3

Cette dèrnière écriture ce n’est que la réduction en carrés de Gauss.

Remarque 4.25 Attention, les éléments de la diagonale de la matrice d’une forme

quadratique dans une base q orthogonale ne sont pas nécessairement les valeurs

propres de M = MB (q):

Corollaire 4.26 Soit q une forme quadratique de signature (s; t) et B une base de

E: Si M = MB (q), alors s est le nombre de valeurs propres positives de M et t est le

nombre de valeurs propres négatives de M:

79
Proposition 4.27 Soit q une forme quadratique dé…nie sur un espace vectoriel réel

E de signature (s; t) et M = matB (q):

1. q est positive ssi toutes les valeurs propres de M sont positives.

2. q est dé…nie positive ssi toutes les valeurs propres de M sont strictement posi-

tives.

4.3.4 Formes quadratiques équivalentes

Dé…nition 4.28 Deux formes quadratiques q1 ; q2 dé…nies sur E sont dites équiva-

lentes s’il existe un automorphisme u : E ! E tel que

q2 = q1 u:

Remarque 4.29 La relation binaire "équivalente" est une relation d’équivalence.

Proposition 4.30 Soient B une base de E; q1 et q2 deux formes quadratiques de E

de matrices A1 et A2 respectivement dans B: Alors, q1 et q2 sont équivalentes si et

seulement s’il existe une matrice inversible P telle que

A2 =t P A1 P:

q1 et q2 sont équivalentes si et seulement s’il existe un automorphisme u : E ! E

tel que

q2 = q1 u:

Notons P = matB (u) ; soient x 2 E et X la matrice colonne des coordonnées de x

dans B: On a

q2 (x) = q1 (u (x)) ,t XA2 X =t (P X) A1 P X =t X t


P A1 P X:

D’où

A2 =t P A1 P:

80
Remarque 4.31 Deux forme quadratiques sont équivalentes si elles ont la même

signature.

Exercise 4.32 Trouver les réductions de Gauss des formes quadratiques suivantes,

en précisant une base orthogonale, le rang et la signature. On précisera pour chaque

forme quadratique, la matrice dans la base canonique de R3 et la forme polaire asso-

ciée :

1. q(x; y; z) = x2 2y 2 + xy + yz

2. q(x) = x21 + 4x22 + 9x23 + 2x1 x2 + 6x2 x3

3. q(x) = x21 + 2x22 + 15x23 4x1 x2 + 6x1 x3 8x2 x3 :

81
Carl Friedrich Gauss 1777 - 1855

Carl Friedrich Gauss, né le 30 avril 1777 à Brunswick et mort le 23 février 1855

à Gottingen, est un mathématicien, astronome et physicien allemand. Il a apporté

de très importantes contributions à ces trois domaines. Surnommé « le prince des

mathématiciens » , il est considéré comme l’un des plus grands mathématiciens de

tous les temps.

La qualité extraordinaire de ses travaux scienti…ques était déjà reconnue par ses

contemporains. Dès 1856, le roi de Hanovre …t graver des pièces commémoratives

avec l’image de Gauss et l’inscription Mathematicorum Principi (« au prince des

mathématiciens » en latin). Gauss n’ayant publié qu’une partie de ses découvertes,

la postérité découvrit surtout l’étendue de ses travaux lors de la publication de ses

oeuvres, de son journal et d’une partie de ses archives, à la …n du xixe siècle.

Gauss dirigea l’Observatoire de Gottingen et ne travailla pas comme professeur

de mathématiques, d’ailleurs il n’aimait guère enseigner, mais il encouragea plusieurs

de ses étudiants, qui devinrent d’importants mathématiciens, notamment Gotthold

Eisenstein et Bernhard Riemann.

82
Chapitre V

FORMES HERMITIENNES

Ce chapitre est l’analogue de l’étude des formes bilinéaires symétriques et formes

quadratiques dans les chapitres 2 et 4 quand on remplace le corps R par C. De façon

simpli…ée : « on remplace le carré x2 d’un nombre réel par le module au carré jzj2 = zz

d’un nombre complexe » . Ceci conduit à la notion de forme hermitienne (analogue

de la notion de forme bilinéaire symétrique), puis de produit scalaire hilbertien sur

Cn (analogue du produit scalaire euclidien sur Rn ). Un des avantages de se placer sur

le corps C est l’existence de valeurs propres et vecteurs propres ; on obtient ainsi les

importants théorèmes de diagonalisation.

5.1 Rappels sur les nombres complexes


Dans tout ce chapitre, le corps de base est C. On note i une racine carrée de 1,

choisie une fois pour toutes. On rappelle les points suivants.

Dé…nition 5.1 (Parties réelle et imaginaire, conjugaison complexe, module et argumen

(1) Tout z 2 C s’écrit de façon unique z = a + ib, avec a; b 2 R ; a s’appelle la par-

tie réelle de z et se note <(z), b s’appelle la partie imaginaire de z et se note =(z).

Remarquons que, comme iz = b ia, on a <( iz) = =(z) et =(iz) = <(z).

(2) La conjugaison complexe est l’application qui a tout z = a+ib associe z = a ib:

Remarquons que :

z + z = 2<(z); z z = 2i=(z)

et

z = z , z 2 R:

83
D’autre part, on a

z + z 0 = z + z 0 ; et zz0 = zz 0 :
p
(3) Si z = a + ib, on a zz = a2 + b2 et jzj = zz s’appelle le module (ou la

norme) de z. D’après la dernière egalité ci-dessus, la norme est multiplicative, i.e. on

a jz1 z2 j = jz1 j : jz2 j.


z
(4) En…n, si z 6= 0, alors jzj
est de module 1, donc de la forme ei , avec 2 R=2 Z.

Donc tout z 6= 0 sӎcrit de fa con unique :

z = ei ; avec = jzj 2 R+ et 2 R dé…ni modulo 2 Z;

par exemple, on peut prendre dans [0; 2 [ ou bien dans ] ; ]) ; on dit que est

l’argument de z.

Dans tout ce chapitre, E est un C espace vectoriel.

5.2 Formes hermitiennes


Dé…nition 5.2 Soit F un C espace vectoriel. On dit qu’une application f : E ! F

est semilinéaire (ou antilinéaire) si et seulement si : pour tout x; y 2 E; 2C

i) f (x + y) = f (x) + f (y)

ii) f ( x) = f (x)

Remarque 5.3 Une forme semilinéaire est une application semilinéaire d’un C espace

vectoriel dans C. C-à-d. si f : E ! C est semilinéaire, alors f est dite forme semili-

néaire.

Exemple 5.4 1. Soit


f :C!C

z 7! z
on a C est un C espace vectoriel et pour tout x; y; 2C

f (x + y) = (x + y) = x + y = f (x) + f (y)

f ( x) = ( x) = :x = f (x)

84
d’où f est semilinéaire.

2. Soit
f : C [X] ! C

P 7! P (z0 ); z0 2 C
On a pour tout P; Q 2 C [X] ; 2C:

f (P + Q) = (P + Q) (z0 ) = P (z0 ) + Q(z0 ) = P (z0 ) + Q (z0 )

= f (P ) + f (Q)

f ( P ) = ( P ) (z0 ) = :P (z0 ) = f (P )

d’où f est semilinéaire.

3. Montrer que l’application

f : C3 ! C; f (x; y; z) = x + 2y z

est antilinéaire.

4. Même question pour

f : Mn (C) ! C; A 7! tr A :

Dé…nition 5.5 On dit qu’une application h : E E ! C est une forme sesquili-

néaire sur E si et seulement si :

i ) h est linéaire par rapport à la première variable

ii ) h est semilinéaire par rapport à la deuxième


8 variable
>
< h ( x + x0 ; y) = h(x; y) + h(x0 ; y)
0 0
C’est-à-dire : 8x; x ; y; y 2 E; 8 2 C :
>
: h (x; y + y 0 ) = h(x; y) + h (x; y 0 )

Remarque 5.6 Pour la dé…nition d’une forme sesquilinéaire, on rencontre parfois

dans la littérature :

i ) h est linéaire par rapport à la deuxième variable.

ii ) h est semilinéaire par rapport à la première variable.

85
Dé…nition 5.7 (Formes hermitiennes) On dit qu’une forme sesquilinéaire h sur

E est une forme hermitienne sur E ssi h a la propriété de " symétrie hermitienne"

c-à-d. pour tout x; y 2 E;

h(y; x) = h(x; y):

On déduit de la symétrie hermitienne de h que pour tout x 2 E; h(x; x) = h(x; x);

d’où h(x; x) 2 R:

Remarque 5.8 1 On note par H(E) l’ensemble des formes hermitienne sur E; si

h; h0 2 H(E) et ; 2 R; on voit facilement que l’application

h + h0 : E E!C

dé…nie par

( h + h0 ) (x; y) = h(x; y) + h0 (x; y)

est encore une forme hermitienne. Par conséquent, H(E) est un R-espace vectoriel (

mais pas un C-espace vectoriel, car si 2C R; alors h ne véri…e pas la propriété

de " symétrie hermitienne" ; h(x; y) 6= h(x; y)).

2 Remarquons que, pour véri…er qu’une application h : E E ! C est une forme

hermitienne, il su¢ t de voir que h véri…e la propriété de " symétrie hermitienne"

et est linéaire en la première variable ; ces deux conditions impliquent en e¤et et la

semi-linéarité en la 2eme variable, car :

h (x; y + y 0 ) = h ( y + y 0 ; x) = h(y; x) + h(y 0 ; x) = h(y; x)+h(y 0 ; x) = h(x; y)+h(x; y 0 ):

Remarque 5.9 La notion de forme hermitienne sur un C espace vectoriel est une

« variante » de la notion de forme bilinéaire sur un R espace vectoriel.

Exemple 5.10 1. Soit


h : C2 ! C

(z1 ; z2 ) 7! z1 z 2

86
Pour tout z1 ; z2 ; z10 ; z20 ; 2 C; on a :

h( z1 + z10 ; z2 ) = ( z1 + z10 ) z 2 = z1 z 2 + z10 z 2 = h (z1 ; z2 ) + h (z10 ; z2 )

h(z1 ; z2 + z20 ) = z1 ( z2 + z20 ) = z1 z2 + z20 = z1 z2 + z1 z20 = h (z1 ; z2 ) +

h (z1 ; z20 ) ; donc h est une forme sesquilinéaire.

h(z2 ; z1 ) = z2 z1 = z2 z1 = z1 z2 = h(z1 ; z2 ): donc h est une forme hermitienne.

2. Soit
h : C2 ! C

(z1 ; z2 ) 7! iz1 z 2
On véri…e de la même façon que h est une forme sesquilinéaire. Mais pout tout z1 ; z2 2

C; on a :

h(z2 ; z1 ) = iz2 z1 = iz2 z1 = iz1 z2 = iz1 z2

= h(z1 ; z2 ):

6= h(z1 ; z2 ):

Donc h n’est pas une forme hermitienne.

Exercise 5.11 1. Les applications suivantes sont-elles des formes sesquilinéaires her-

mitiennes ?

(a) h1 : C3 C3 ! C donnée par :

h1 (x; y) = x1 y1 + 3x2 y 2 + 2ix3 y3 + (2 + 3i) x1 y2 +

(2 3i) x2 y1 + (1 5i) x2 y3 + (1 + 5i) x3 y2 :

(b) h2 : Mn (C) Mn (C) ! C donnée par

h2 (A; B) = tr t A:B :

2. Soit E l’espace des fonctions continues de [0; 1] vers C. Pour f 2 E et f = a+ib

avec a; b deux fonctions à valeurs réelles, on pose


Z1 Z1 Z1
f (x)dx = a(x)dx + i b(x)dx:
0 0 0

87
Montrer que

h3 : E E!C
Z1
h3 (f; g) = f (x):g(x)dx
0

est une forme sesquilinéaire hermitienne.

Solution :

1. (a) Pour tout x; y; z 2 C3 et tout 2 C on peut montrer que :

h1 ( x + z; y) = ( x1 + z1 ) y1 + 3 ( x2 + z2 ) y 2 + 2i ( x3 + z3 ) y3 + (2 + 3i) ( x1 + z1 ) y2

+ (2 3i) ( x2 + z2 ) y1 + (1 5i) ( x2 + z2 ) y3 + (1 + 5i) ( x3 + z3 ) y2 :

= h1 (x; y) + h1 (z; y)

De même on montre que :

h1 (x; y + z) = x1 ( y1 + z1 ) + 3x2 ( y2 + z2 ) + 2ix3 ( y3 + z3 ) + (2 + 3i) x1 ( y2 + z2 )

+ (2 3i) x2 ( y1 + z1 ) + (1 5i) x2 ( y3 + z3 ) + (1 + 5i) x3 ( y2 + z2 ):

= h1 (x; y) + h1 (x; z) :

D’où h1 est une forme sesquilinéaire. D’autre part on a :

h1 (y; x) = y1 x1 + 3y2 x2 + 2iy3 x3 + (2 + 3i) y1 x2 +

(2 3i) y2 x1 + (1 5i) y2 x3 + (1 + 5i) y3 x2 :

h1 (x; y) = x1 y1 + 3x2 y 2 + 2ix3 y3 + (2 + 3i) x1 y2 +

(2 3i) x2 y1 + (1 5i) x2 y3 + (1 + 5i) x3 y2

= y1 x1 + 3y2 x2 2iy3 x3 + (2 3i) y2 x1 +

(2 + 3i) y1 x2 + (1 + 5i) y3 x2 + (1 5i) y2 x3 :

6= h1 (y; x)

88
D’où h1 est une forme sesquilinéaire non hermitienne.

(b) Pour tout A; B; C 2 Mn (C) et tout 2C

t
h2 ( A + C; B) = tr ( A + C) :B
t
= tr A:B +t C:B

= tr t A:B + tr t C:B

= h2 (A; B) + h2 (C; B);

et

h2 (A; B + C) = tr t A: B+C

= tr :t AB +t AC

= tr t AB + tr t AC

= h2 (A; B) + h2 (A; C) :

D’où h2 est une forme sesquilinéaire. D’autre part on a :

h2 (B; A) = tr t B:A = trt t B:A

= tr t A:B ;

et

h2 (A; B) = tr t A:B

= tr t A:B

= tr t A:B :

Alors, h2 (B; A) = h2 (A; B). D’où h2 est une forme sesquilinéaire hermitienne.

89
2. Pour tout f; g; k 2 E et tout 2C
Z1
h3 ( f + k; g) = ( f + k) (x):g(x)dx
0
Z1 Z1
= f (x)g(x)dx + k(x)g(x)dx
0 0
= h3 (f; g) + h3 (k; g) :

De même on montre que :


Z1
h3 (f; g + k) = f (x):( g + k) (x)dx
0
Z1
= f (x) g(x) + k(x) dx
0
Z1 Z1
= f (x)g(x)dx + f (x)k(x)dx
0 0

= h3 (f; g) + h3 (f; k) :

D’où h2 est une forme sesquilinéaire. D’autre part on a :


Z1
h3 (g; f ) = g(x)f (x)dx;
0
et
Z1
h3 (f; g) = f (x):g(x)dx
0
Z1
= f (x)g(x)dx
0
= h3 (g; f ) :

D’où h3 est une forme sesquilinéaire hermitienne.

Dé…nition 5.12 (Formes quadratiques hermitiennes) Soit h une forme hermi-

tienne sur un C espace vectoriel E. On dit que l’application

Q : E ! R; Q (x) = h(x; x)

90
est une forme quadratique hermitienne sur E. D’après le lemme qui suit, h est

entièrement déterminée par Q, et l’on dit que h est la forme polaire de Q. Notons

aussi que pour tout 2 C, on a

Q( x) = h ( x; x) = h (x; x) = h (x; x) = Q(x) = j j2 Q(x);

où j j est le module ( ou la norme) de 2 C:

Lemme 5.13 (Polarisation) Soient E un C espace vectoriel, h 2 H(E) et Q l’ap-

plication E ! R; x 7! h(x; x). Alors, pour tout x; y 2 E, on a :

(1) < (h(x; y)) = 21 (Q(x + y) Q(x) Q(y)) = 41 (Q(x + y) Q(x y)) :

(2) = (h(x; y)) = 21 (Q(x + iy) Q(x) Q(y)) = 14 (Q(x + iy) Q(x iy)) :

(3) 4h(x; y) = Q(x + y) Q(x y) + iQ(x + iy) iQ(x iy):

Preuve. 1 On a

Q(x + y) = h (x + y; x + y) = Q(x) + Q(y) + h(x; y) + h(y; x)

Q(x y) = h (x y; x y) = Q(x) + Q(y) h(x; y) h(y; x)

et comme

h(x; y) + h(y; x) = h(x; y) + h(x; y) = 2< (h(x; y)) ;

on obtient (1) : Comme =(z) = <( iz) pour tout z 2 C, on obtient

=(h(x; y)) = <( ih(x; y)) = < (h(x; iy))

et donc (2) s’obtient en remplaçant y par iy dans (1) et en utilisant que Q(iy) =

jij2 Q(y) = Q(y): En…n, (3) découle de (1) et (2).

Désormais, on suppose E de dimension …nie n.

Dé…nition 5.14 (Matrices hermitiennes) Une matrice A 2 Mn (C) est hermi-

tienne si t A = A. On note M Hn (C) l’ensemble de ces matrices, si A; B 2 M Hn (C)

et s; t 2 R, alors sA + tB 2 M Hn (C), donc M Hn (C) est un R espace vectoriel

91
(mais pas un C espace vectoriel). Observons que si A 2 M Hn (C), ses coe¢ cients

diagonaux aii véri…ent aii = aii donc aii 2 R:

Dé…nition 5.15 (Matrice d’une forme hermitienne) Soit h une forme hermi-

tienne sur un C espace vectoriel E de dimension n et soit B = (e1 ; :::; en ) une base

de E.

La matrice M atB (h) de h dans la base B est la matrice A = (aij )1 i;j n 2 Mn (C),

où aij = h(ei ; ej ). Comme h (ej ; ei ) = h (ei ; ej ) on a aji = aij , donc t A = A, i.e.

A 2 M Hn (C).

h est entièrement déterminée par sa matrice A : en e¤et, d’après la linéarité (resp.

semi-linéarité) en la 1ère (resp. 2 ème) variable, on a l’égalité :


!
Xn X
n
8x; y 2 E; h(x; y) = h xi ei ; yj ej
i=1 j=1

X
n X
n
= xi yj h (ei ; ej ) = aij xi yj : (*)
i;j=1 i;j=1

Donc, si l’on note X; Y 0


les vecteurs
1 colonnes
0 1des coordonnées des vecteurs x; y res-
B x1 C B y1 C
B . C B C
pectivement c-à-d. X = B .. C ; Y = B ... C 2 Cn ; on a la formule matricielle
B C B C
@ A @ A
xn yn

h(x; y) =t XAY :

Réciproquement, pour tout A = (aij )1 i;j n 2 Mn (C), l’application hA : E E ! C


Pn Pn Pn
dé…nie par hA i=1 xi ei ; j=1 yj ej = i;j=1 aij xi yj est une forme hermitienne sur

E, et M atB (hA ) = A. Donc, se donner une forme hermitienne sur E « est la même

chose » que se donner une matrice hermitienne : de façon précise, l’application

B : H(E) ! M Hn (C)

h 7! M atB (h)

est un isomorphisme de R espaces vectoriels.

92
Théorème 5.16 (Changement de base) Soient h une forme hermitienne sur un

C espace vectoriel E de dimension n; B = (e1 ; :::; en ) une base de E telle que A =

M atB (h). Si B 0 est une autre base de E et P la matrice de passage M atB (B 0 ). Alors

A0 = M atB 0 (h) =t P AP :

Preuve. Soient x; y 2 E, ils correspondent dans la base B (resp. B 0 ) à des vecteurs

colonnes X; Y (resp. X 0 ; Y 0 ). D’après la formule de changement de coordonnées, on a

X = P X 0 et Y = P Y 0 , d’où t X =t X 0 :t P et Y = P Y 0 , et donc

h(x; y) =t XAY =t X 0 t P AP Y 0

ce qui entraîne A0 =t P AP .

Dé…nition 5.17 (Expression d’une forme quadratique hermitienne) En sépa-

rant, d’une part, les termes xi yi et, d’autre part, les termes xi yj avec i 6= j; la formule

(*) se récrit de la façon suivante (puisque aji = aij pour tout i 6= j) :

X
n X
h (x; y) = aii xi yi + (aij xi yj + aij xj yi ) :
i=1 1 i<j n

En particulier, prenant y = x (i:e: yi = xi pour tout i), on voit que la forme qua-

dratique hermitienne Q associée à h est donnée par la formule suivante (noter que

xi xi = jxi j2 ) :

X
n X X
n X
Q(x) = aii jxi j2 + (aij xi xj + aij xi xj ) = aii jxi j2 + < (aij xi xj ) :
i=1 1 i<j n i=1 1 i<j n

On voit donc apparaîre les carrés des modules des xi , et les parties réelles des doubles

produits xi xj . Pour abréger, on parlera de « carrés de modules » et de « doubles

produits » .

Dé…nition 5.18 (Rang d’une forme hermitienne) Soit h une forme hermitienne

sur un C espace vectoriel E de dimension n et soit B = (e1 ; :::; en ) une base de E.

93
(1) On dé…nit le rang de h par rang(h) = rang(A), où A = M atB (h) ; ceci ne

dépend pas du choix de la base B.

(2) On dit que h est non-dégénérée si rang(h) = dimE, i.e. si sa matrice dans

une (et donc dans toute) base de E est inversible.

Dé…nition 5.19 (Orthogonalité) Soit h une forme hermitienne sur un C espace

vectoriel E.

(1) On dit que deux vecteurs x; y 2 E sont orthogonaux (pour h) si h(x; y) = 0 ;

ceci équivaut à dire que h(y; x) = 0 (puisque h(y; x) = h(x; y) et vice-versa). Plus

généralement, on dit que deux sous-ensembles X; Y de E sont orthogonaux si l’on a

h(x; y) = 0 pour tout x 2 X et y 2 Y . On notera X ? Y pour signi…er que X et Y

sont orthogonaux.

(2) Pour tout sous-ensemble Y de E, on dé…nit son orthogonal (relativement a h


?
), noté Y par :

Y ? = fx 2 E j h(x; y) = 0; 8y 2 Y g

c’est un sous-espace vectoriel de E (même si Y n’en est pas un) ; de plus, on a les

propriétés suivantes :

Y Z ) Z? Y ?; Y ? = V ect (Y )?

en particulier, si Y = F est un sous-espace vectoriel de E et si (f1 ; :::; fp ) est une

famille génératrice de F , alors

F ? = ff1 ; :::; fp g? = fx 2 E j h(x; fi ) = 0; 8i = 1; :::; pg :

(3) On pose

ker (h) = fx 2 E j h(x; y) = 0; 8y 2 Eg

et on l’appelle le noyau de h. On dit que h est non-dégénérée si ker (h) = f0g.

94
Théorème 5.20 (Orthogonal d’un sous-espace) Soit h une forme hermitienne

sur un C espace vectoriel E de dimension n et soit F un sous-espace vectoriel de E,

de dimension r.
?
(1) On a F F? et dim F ? dim E dim F:

(2) ker (h) = f0g , h est non-dégénérée.


?
(3) Si h est non-dégénérée, on a dim F ? = dim E dim F et F = F ? .

(4) Si F \ F ? = f0g, alors E = F F ?:

Preuve. (1) Soit f 2 F , pour tout x 2 F ? on a h(f ; x) = h(x; f ) = 0, d’où

f 2 (F ? )? . Ceci montre la premi ere assertion (1).

Soit (f1 ; :::; fr ) une base de F , complétons-la en une base B = (f1 ; :::; fn ) de E,

et soit A = (aij )1 i;j n la matrice de h dans la base B, i.e. aij = h(fi ; fj ) pour

i; j = 1; :::; n:D’après le point (2) de la dé…nition 5.19, F ? est formé des vecteurs

v = x1 f1 + ::: + xn fn 2 E tels que h(v; fi ) = 0 pour i = 1; :::; r. Comme h(x1 f1 + ::: +


P P
xn fn ; fi ) = nj=1 xj h (fj ; fi ) = nj=1 aji xj , ceci équivaut à dire que le vecteur colonne
0 1
B x1 C
B . C
X=B . C
B . C est solution du système linéaire homogène
@ A
xn
8
>
> a11 x1 + ::: + an1 xn = 0
>
>
<
..
> .
>
>
>
: a1r x1 + ::: + anr xn = 0

dont la matrice M est formée des r premières lignes de la matrice t A. Comme l’espace

des solutions du système est de dimension n rang(M ), on obtient

dim F ? = n rang(M ) n r;

ce qui prouve la seconde assertion de (1).

(2) Dans le cas particulier où F = E, on a M =t A et, comme rang(t A) = rang(A),

on obtient que dimE ? = n rang(A). Donc ker (h) = E ? est nul si et seulement si

95
rang(A) = n. Ceci prouve (2).

(3) Supposons h non-dégénérée. Alors A est de rang n, i.e. ses colonnes sont

linéairement indépendantes, en particulier les r premières colonnes le sont, donc la

matrice M est de rang r, et donc dim F ? = n r. Remplaçant alors F par F ? , on


?
obtient l’égalité dim(F ? )? = n (n r) = r, et par conséquent l’inclusion F F?

est une égalité. Ceci prouve (3).

(4) En…n, supposons F \F ? = f0g (sans supposer h non-dégénérée). Alors F et F ?

sont en somme directe, et le sous-espace F F ? de E est de dimension d = r+dimF ? .

D’après (1), on a d n, d’où E = F F ? (et dimF ? = n r). Ceci prouve (4).

Dé…nition 5.21 (Bases orthogonales) Soit E un C espace vectoriel de dimen-

sion n et soient h une forme hermitienne sur E, et Q la forme quadratique hermi-

tienne associée. Soit B = (e1 ; :::; en ) une base de E

(1) On dit que B est une base orthogonale pour h (ou pour Q) si h (ei ; ej ) = 0

pour i 6= j.

(2) Ceci équivaut à dire que la matrice A = M atB (h) est diagonale ; si l’on note

1 ; :::; n ses coe…cients diagonaux (qui sont réels) et (z1 ; :::; zn ) les coordonnées dans

la base B, ceci équivaut encore à dire que

Q(z1 ; :::; zn ) = 1 jz1 j2 + ::: + n jzn j2 :

Théorème 5.22 (de Sylvester dans le cas hermitien) Soit h une forme hermi-

tienne sur un C espace vectoriel E de dimension n, et soit Q la forme quadratique

hermitienne associée.

(1) Il existe une base B de E orthogonale pour h.

(2) Soient B = (e1 ; :::; en ) une base orthogonale pour h et D la matrice diagonale

M atB (h). Quitte à renuméroter les ei , on peut supposer que les coe¢ cients diago-

naux 1 ; :::; r 2 R sont non nuls et que i = 0 pour i > r. Notons (z1 ; :::; zn ) les

coordonnées dans la base B, alors :

96
(a) On a Q(z1 ; :::; zn ) = 1 jz1 j2 + ::: + r jzr j2 :

(b) Soit p (resp. q) le nombre d’indices i tels que Q(ei ) > 0 (resp. < 0). Alors

p et q ne dépendent pas de la base orthogonale choisie.

(c) Le couple (p; q) s’appelle la signature de h ; on a p + q = r = rang(h).

(d) ker(h) est le sous-espace V ect(er+1 ; :::; en ), donné par les equations

z1 = 0 = ::: = zr :

(3) De plus, on peut choisir B de sorte que la matrice diagonale D = M atB (h)

ait pour termes diagonaux (1; :::; 1; 1; :::; 1; :::; 0; :::; 0), le nombre de 1 (resp. 1)

etant p (resp. q).

Preuve. (1) Montrons l’existence d’une base orthogonale en procédant par récur-

rence sur n = dimE. Il n’y a rien à montrer si n = 0 ou si h = 0. On peut donc

supposer n 1, le résultat etabli pour n 1, et h 6= 0. Alors, d’après le lemme

5.13, la forme quadratique hermitienne Q est non nulle, donc il existe e1 2 E tel que

Q(e1 ) 6= 0. Posons F = ke1 , comme h(e1 ; e1 ) 6= 0, alors F \ F ? = f0g donc, d’après

le théorème 5.20, on a

E=F F ?:

Par hypothèse de récurrence, il existe une base (e2 ; :::; en ) de F ? telle que h(ei ; ej ) = 0

pour i 6= j. Alors (e1 ; e2 ; :::; en ) est une base de E orthogonale pour h. Ceci prouve

l’assertion (1).

(2) (2.a) et l’égalité p + q = r = rang(h) dans (2.c) découlent aussitôt des dé…ni-

tions. Prouvons maintenant (2.d). D’après (*), h est donnée dans la base B par
X
n X
n
8u = xi ei ; 8v = yj ej ; h(u; v) = 1 x1 y 1 + ::: + r xr y r : (*’)
i=1 j=1

Supposons u 2 ker (h), alors pour tout i = 1; :::; r, prenant v = ei (c.- a-d., yi = 1 et

yj = 0 pour j 6= i), on obtient xi = 0; d’où u 2 F = V ect(er+1 ; :::; en ). Réciproque-

ment, (*’) montre aussi que tout u 2 F (i.e. tel que x1 = 0 = ::: = xr ) appartient a

ker(h) d’où l’égalité désirée. Ceci prouve (2.d).

97
Prouvons (2.b). On note r = rang(h). Soient B = (e1 ; :::; en ) et C = (f1 ; :::; fn )

deux bases de E orthogonales pour h. Notons p (resp. p0 ) le nombre d’indices i tels

que Q(ei ) > 0 (resp. Q(fi ) > 0) et q (resp. q 0 ) le nombre d’indices i tels que Q(ei ) < 0

(resp. Q(fi ) < 0). Alors

p + q = r = p0 + q 0

et il s’agit de montrer que q = q 0 et p = p0 . Quitte a renuméroter les éléments de B

et C , on peut supposer que


8 8
>
> Q(ei ) > 0 pour i = 1; :::; p >
> Q(fi ) > 0 pour i = 1; :::; p0
>
> >
>
< <
> Q(ei ) < 0 pour i = p + 1; :::; p + q ; > Q(fi ) < 0 pour i = p0 + 1; :::; p0 + q 0
>
> >
>
>
: Q(ei ) = 0 pour i > p + q = r >
: Q(fi ) = 0 pour i > p0 + q 0 = r:
( )

Notons P+ le sous-espace de E engendré par les vecteurs ei tels que Q(ei ) 0. Ces

vecteurs sont au nombre de n q, donc dimP+ = n q. Soit x un élément arbitraire


P
de P+ , ecrivons x = i2I xi ei , avec I = f1; :::; pg [ fr + 1; :::; ng ; alors, d’après ( ),

on obtient
p
X
Q(x) = jxi j2 Q(ei ) 0:
i=1

D’autre part, soit P 0 le sous-espace de E engendré par les vecteurs fj tels que Q(fj ) <

0. Ces vecteurs sont au nombre de q 0 , donc dimP 0 = q 0 . Soit y un élément non nul de
P 0 +q0
P 0 ; on peut ecrire y = pj=p 0 +1 yj fj , avec au moins l’
un des yj non nul (car y 6= 0).

Alors, d’après ( ) à nouveau, on obtient


p0 +q 0
X
Q(y) = jyjj2 Q(fj ) < 0:
j=p0 +1

Par conséquent, on a P+ \ P 0 = f0g et donc

n = dimE dimP+ + dimP 0 = n q + q0

d’où q q 0 . Echangeant les rôles des bases B et C, on obtient de même q0 q, d’où

q = q 0 , et de même p = p0. Ceci prouve (2.b).

98
(3) Soit B = (e1 ; :::; en ) comme ci-dessus ; pour i = 1; :::; p+q, notons jQ(ei )j > 0 la
p
valeur absolue du réel Q(ei ) 6= 0. En remplaçant ei par ei jQ(ei )j, pour i = 1; :::; p+q,

on obtient une base orthogonale ayant la propriété énoncée dans (3). Ceci achève la

démonstration du thérème.

Exercise 5.23 E est un espace vectoriel sur C .

1) Parmi les application suivantes de E E vers C, déterminer lesquelles sont :

Sesquilinéaires, Hermitiennes.

f : E E!C

(a; b) 7! (1 + i) a1 b1 + 2a1 b2

g : E E!C

(a; b) 7! a1 b1 + (1 2i) a2 b1 + (1 + 2i) a1 b2

avec E = C2 ; a = (a1 ; a2 ) et b = (b1 ; b2 ) :

2) Donner la matrice de g dans la base canonique de C2 .

3) Montrer que g est nondégénérée.

Solution : 1) On peut véri…er facilement que


8
>
< f ( a + a0 ; b) = f (a; b) + f (a0 ; b)
0 0
8a; a ; b; b 2 E; 8 2 C : ;
>
: f (a; b + b0 ) = f (a; b) + f (a; b0 )

donc f est sesquilinéaire. D’autre part

f (b; a) = (1 + i) b1 a1 + 2b1 a2 6= (1 + i) a1 b1 + 2a1 b2 = f (a; b);

d’où f est une forme sesquilinéaire non hermitienne.

99
Solution 5.24 De même on peut véri…er facilement que g est sesquilinéaire. D’autre

part

g(b; a) = b1 a1 + (1 2i) b2 a1 + (1 + 2i) b1 a2

= a1 b1 + (1 2i) a2 b1 + (1 + 2i) a1 b2

= g(a; b);

d’où g est une forme sesquilinéaire hermitienne.

2) On a

g(a; b) = a1 b1 + (1 2i) a2 b1 + (1 + 2i) a1 b2

= a1 b1 + (1 + 2i) b2 + a2 (1 2i) b1
0 1
B b1 + (1 + 2i) b2 C
= (a1 a2 ) @ A
(1 2i) b1
0 10 1
B 1 1 + 2i C B b1 C
= (a1 a2 ) @ A@ A
1 2i 0 b2
0 1 0 1
B a1 C B b1 C
= t (A) M B; ou A = @ A ;B = @ A;
a2 b2
0 1
B 1 1 + 2i C
donc la matrice de g est M = @ A:
1 2i 0
3) On remarque que : det (M ) = 5 6= 0, donc rg (M ) = 2 = dim E, d’où g est

nondégénérée.

Exercise 5.25 Soit E un espace vectoriel complexe de dimension 2 et f une appli-

cation dé…nie par :

f : E E!C

(x; y) 7! 3x1 y1 + (1 + i) x2 y2 ix1 y2 + (2 i) x2 y1

1) Montrer que f est une forme hermitienne.

100
2) Donner la matrice de f dans la base canonique de E:

3) Montrer que f est nondégénérée.

101
RÉFÉRENCES

[1] E. Azoulay et J. Avignant. Mathématiques, tome 4. Algébre. Cours et Exercices.

Copyright c 1984 by McGraw-Hill, Paris.

[2] V. Prasolov. Problems and theorems in linear algebra.

[3] Khalid Koufany. Cours d’algèbre bilinéaire, Université de Lorraine, version 1, jan-

vier 2017.

[4] Jean-Marie Monier. Algèbre MP- Cours et 500 exercices corrigés, 4e édition, DU-

NOD, 2004.

[5] Richard Gomez. Diagonalisation des matrices symétriques, article publié sur Mé-

gamaths, 2007.

[6] Lionel Schwartz. Algébre 3e année, Cours et exercices avec solutions. c Dunod,

Paris, 2003.

[7] Joseph Grifone. Algébre linéaire, 4ème édition. Cépaduès Edition, France, 2011.

[8] Stéphane Balac et Laurent Chupin, Analyse et algèbre, Cours de mathématiques

de 2eme année avec exercices corrigés. Presses polytechniques et universitaires

romandes, 2008.

102

Vous aimerez peut-être aussi